Буквально на днях, после прочтения статьи Путешествия во времени и программирование я загорелся идеей об экспериментальных исследованиях, которые позволили бы получить практические ответы на вопросы о перемещении во времени. Но прежде чем переходить к экспериментам, требуется разработать теоретическое обоснование о возможности преодоления времени между прошлым и будущим. Чем собственно я занимался в течении последних дней. Исследование основано на теории относительности Эйнштейна и релятивистских эффектах, попутно затрагивая также квантовую механику и теорию суперструн. Думаю мне удалось получить положительные ответы на поставленные вопросы, подробно рассмотреть скрытые измерения и попутно получить объяснение некоторых явлений, например, природу корпускулярно-волнового дуализма. А также рассмотреть практические способы передачи информации между настоящим и будущем. Если вас тоже волнуют эти вопросы то добро пожаловать под кат.

Обычно я не занимаюсь теоретической физикой, и в реальности веду довольно однообразную жизнь занимаясь софтом, железом, и отвечая на однотипные вопросы пользователей. Поэтому если найдутся неточности и ошибки надеюсь на конструктивное обсуждение в комментариях. Но мимо данной темы я не смог пройти. В голове то и дело появлялись новые идеи, которые со временем образовались в единую теорию. Я как то не рвусь самому отправляться в прошлое или будущее в котором меня никто не ожидает. Но предполагаю, что в будущем это станет возможно. Меня больше интересуют решение прикладных задач связанных с созданием информационных каналов для передачи информации между прошлым и будущем. А также волнуют вопросы о возможности изменения прошлого и будущего.

Путешествие в прошлое связано с большим количеством трудностей, которые сильно ограничивают возможность такого путешествия. На данном этапе развития науки и техники, думаю преждевременно браться за реализацию таких идей. Но прежде чем понять, можем ли мы изменить прошлое, необходимо определиться с тем, можем ли изменить настоящее и будущее. Ведь суть любых изменений прошлого сводится к изменению последующих событий относительно заданной точки времени, к которому мы хотим вернуться. Если в качестве заданной точки взять текущий момент времени, то необходимость перемещения в прошлое отпадает, также как отпадает большое количество трудностей связанных с таким перемещением. Остается только узнать цепь событий, которые должны произойти в будущем, и попытаться разорвать эту цепь, чтобы получить альтернативное развитие будущего. На самом деле, нам даже не нужно знать полную цепочку событий. Необходимо достоверно узнать сбудется или нет одно конкретное событие в будущем (которое будет объектом исследования). Если сбудется, то значит, цепь событий привело к тому, чтобы это событие сбылось. Тогда у нас появляется возможность повлиять на ход эксперимента и сделать так, чтобы это событие не сбылось. Получится ли нам это сделать вопрос пока не ясный. И дело не в том, сможем ли мы это сделать (экспериментальная установка должна позволить это сделать), а в том, возможно ли альтернативное развитие реальности.

В первую очередь возникает вопрос — как можно достоверно узнать то, что еще не случилось? Ведь все наши знания о будущем всегда сводятся только к прогнозам, а для подобных экспериментов прогнозы не годятся. Полученные в ходе эксперимента данные должны неопровержимо доказывать то, что должно произойти в будущем, как о уже произошедшем событии. Но на самом деле есть способ получения таких достоверных данных. Если как следует рассмотреть теорию вероятности Эйнштейна и квантовую механику, то можно найти такую частицу, которая сможет связать прошлое и будущее в одну линию времени и передать нам необходимую информацию. В качестве такой частицы выступает фотон.

Суть эксперимента сводится к знаменитому опыту с двумя щелями с отложенным выбором, который был предложен в 1980 г. физиком Джоном Уилером. Есть много вариантов реализации такого эксперимента, одно из которых приводилось на Хабре. В качестве примера рассмотрим эксперимент с отложенным выбором, который был предложен Скалли и Дрюлем:


На пути источника фотонов — лазера — ставят светоделитель, в качестве которого выступает полупрозрачное зеркало. Обычно такое зеркало отражает половину падающего на него света, а другая половина проходит насквозь. Но фотоны, будучи в состоянии квантовой неопределенности, попадая на светоделитель будут выбирать оба направления одновременно.

После прохождения светоделителя фотоны попадают в даун-конверторы. Даун-конвертор — это прибор, который получает один фотон на входе и производит два фотона на выходе, каждый с половиной энергии («даун-преобразование») от исходного. Один из двух фотонов (так называемый сигнальный фотон) направляется вдоль исходного пути. Другой фотон, произведённый даун-конвертором (именуемый холостым фотоном), посылается в совершенно другом направлении.

Используя полностью отражающие зеркала, расположенные по бокам, два луча снова собираются вместе и направляются к детекторному экрану. Рассматривая свет в виде волны, как в описании Максвелла, на экране можно видеть интерференционную картину.

В эксперименте можно определить какой путь к экрану выбрал сигнальный фотон, путём наблюдения, который из даун-конверторов испустил холостой фотон-партнёр. Так как есть возможность получить информацию о выборе пути сигнального фотона (даже хотя она является полностью косвенной, поскольку не взаимодействуем ни с одним сигнальным фотоном) — наблюдение за холостым фотоном вызывает предотвращение возникновения интерференционной картины.

Итак. Причем тут опыты с двумя щелями


Дело в том, что холостые фотоны, испускаемые даун-конверторами, могут проходить гораздо большее расстояние, чем их сигнальные фотоны-партнёры. Но какое бы расстояние не прошли холостые фотоны, картина на экране всегда будет совпадать с тем, будут ли холостые фотоны зафиксированы или нет.

Допустим, что расстояние холостого фотона до наблюдателя во много раз превышает, чем расстояние сигнального фотона до экрана. Получается, что картина на экране будет заранее отображать тот факт, будут ли наблюдать за холостым фотоном-партнёром или нет. Если даже решение о наблюдение за холостым фотоном принимает генератор случайных событий.

Расстояние, которое может пройти холостой фотон, никак не влияет на результат, который отображается на экране. Если загнать такой фотон в ловушку и, например, заставить многократно крутиться по кольцу, то можно растянуть данный эксперимент на произвольно долгое время. Не зависимо от продолжительности эксперимента мы будем иметь достоверно установленный факт того, что должно случиться в будущем. Например, если решение о том, будем ли мы «ловить» холостой фотон зависит от подбрасывания монеты, то уже в начале эксперимента мы будем знать, «каким образом упадет монетка». Когда на экране появиться картинка, это будет уже свершившийся факт еще до подбрасывания монеты.

Возникает интересная особенность, которая кажется меняет причинно-следственную связь. Мы можем спросить – каким образом следствие (которое произошло в прошлом) может формировать причину (которое должно произойти в будущем)? А если причина еще не наступала, то каким образом мы можем наблюдать следствие? Чтобы это понять попробуем углубиться в специальную теорию относительности Эйнштейна и разобраться с тем, что происходит на самом деле. Но в этом случае нам придется рассматривать фотон как частицу, чтобы не смешивать квантовую неопределенность с теорией относительности.

Почему именно фотон


Это именно та частица, которая идеально подходит для данного эксперимента. Конечно, квантовой неопределенностью обладают и другие частицы, такие как электроны и даже атомы. Но именно фотон имеет предельную скорость движения в пространстве и для него не существует само понятие времени, поэтому оно может беспрепятственно пересекать временное измерение, связывая прошлое с будущем.

Картина времени


Чтобы представить время, необходимо рассмотреть пространство-время в виде непрерывного блока растянутого во времени. Срезы, формирующие блок, являются моментами настоящего времени для наблюдателя. Каждый срез представляет пространство в один момент времени с его точки зрения. Этот момент включает в себя все точки пространства и все события во вселенной, которые представляются для наблюдателя как происходящее одновременно. Объединяя эти срезы настоящего, расположив одну за другим в том порядке, в котором наблюдатель переживает эти временные слои, мы получим область пространства-времени.


Но в зависимости от скорости движения, срезы настоящего будут делить пространство-время под разными углами. Чем больше скорость движения относительно других объектов, тем больше получается угол среза. Это означает, настоящее время движущегося объекта не совпадает с настоящим временем других объектов, относительно которых оно движется.


По направлению движению, срез настоящего времени объекта смещается в будущее относительно неподвижных объектов. В обратном направлении движения, срез настоящего времени объекта смещается в прошлое относительно неподвижных объектов. Это происходит потому, как свет, летящий на встречу движущегося объекта достигает его раньше, чем свет, догоняющей движущийся объект с противоположный стороны. Максимальная скорость движения в пространстве обеспечивает максимальный угол смещения текущего момента времени. Для скорости света этот угол составляет 45°.


Замедление времени


Как я уже писал, для частицы света (фотона) не существует понятие времени. Попробуем рассмотреть причину этого явления. Согласно специальной теории относительности Эйнштейна по мере увеличения скорости объекта происходит замедление времени. Это связано с тем, что по мере увеличения скорости движущегося объекта для света требуется преодолевать все большее расстояние за единицу времени. Например, при движении автомобиля, свету его фар необходимо преодолевать большее расстояние за единицу времени, чем если бы автомобиль стоял на парковке. Но скорость света является предельной величиной и не может увеличиваться. Поэтому складывание скорости света со скоростью движения автомобиля не приводит к увеличению скорости света, а приводит к замедлению времени, согласно формуле:

где r – длительность времени, v – относительная скорость движения объекта.
Для наглядности рассмотрим еще один пример. Возьмем два зеркала и расположим их противоположно одну над другой. Допустим, что луч света будет многократно отражаться между этими двумя зеркалами. Движение луча света будет происходить по вертикальной оси, при каждом отражении отмеряя время как метроном. Теперь начнем двигать наши зеркала по горизонтальной оси. С увеличением скорости движения, траектория движения света будет наклоняться по диагонали, описывая зигзагообразное движение.



Чем больше скорость движения по горизонтали, тем сильнее будет наклонена траектория движения луча. При достижении скорости света рассматриваемая траектория движения будет выпрямлена в одну линию, как если бы мы растянули пружину. То есть свет уже перестанет отражаться между двумя зеркалами и будет двигаться параллельно горизонтальной оси. А значит наш «метроном» перестанет отмерять ход времени.

Поэтому для света не существует измерения времени. Фотон не имеет ни прошлого, ни будущего. Для него есть только текущий момент, в котором оно существует.

Сжатие пространства


Теперь попробуем разобраться с тем, что происходит с пространством на скорости света, в котором пребывают фотоны.

Для примера возьмем некий объект длиной в 1 метр и будем ускорять его до около световой скорости. По мере увеличения скорости объекта мы будем наблюдать релятивистское сокращение длины движущегося объекта, согласно формуле:

где l – это длина, а v – относительная скорость движения объекта.

Под словом «мы будем наблюдать» я имею ввиду неподвижного наблюдателя со стороны. Хотя с точки зрения движущегося объекта, неподвижные наблюдатели так же будут сокращаться в длине, ибо наблюдатели будут с той же скоростью двигаться в противоположном направлении относительно самого объекта. Отметим, что длина объекта является измеряемой величиной, а пространство является точкой отсчета для измерения этой величины. Мы также знаем, что длина объекта имеет фиксированную величину в 1 метр и не может меняться относительно пространства, в котором оно измерено. Значит, наблюдаемое релятивистское сокращение длины говорит о том, что сокращается пространство.

Что произойдет, если объект постепенно ускориться до скорости света? На самом деле ни одна материя не может ускоряться до скорости света. Можно максимально приближаться к этой скорости, но достичь скорости света не возможно. Поэтому с точки зрения наблюдателя, длина движущегося объект будет бесконечно сокращаться, пока не достигнет минимально возможной длины. А с точки зрения движущегося объекта, все относительно неподвижные объекты в пространстве будут бесконечно сжиматься, пока не сократятся до минимально возможной длины. Согласно специальной теории относительности Эйнштейна мы также знаем одну интересную особенность — не зависимо от скорости движения самого объекта, скорость света всегда остается неизменной предельной величиной. Значит, для частицы света всё наше пространство сжато до размеров самого фотона. Причем сжаты все объекты, не зависимо от того двигаются они в пространстве или остаются неподвижными.

Тут можно заметить, что формула релятивистского сокращения длины недвусмысленно дает нам понять, что при скорости света всё пространство будет сжато до нулевого размера. Я же писал о том, что пространство будет сжато размеров самого фотона. Полагаю, оба вывода являются правильными. С точки зрения Стандартной модели фотон является калибровочным бозоном, выполняющую роль переносчика фундаментальных взаимодействий природы, для описания которого требуется калибровочная инвариантность. С точки зрения М-теории, которая на сегодняшний день претендует на звание Единой теории всего, считается, что фотон представляет из себя колебание одномерной струны со свободными концами, которая не имеет размерности в пространстве и может содержать в себе свернутые измерения. Я честно не знаю по каким расчетам сторонники теории суперструн пришли к подобным выводам. Но то, что наши расчеты ведут нас к тем же результатам думаю говорит о том, что мы смотрим в правильном направлении. Расчеты теории суперструн перепроверялись десятилетиями.

Итак. К чему же мы пришли:

  1. С точки зрения наблюдателя, всё пространство фотона свернуто до размеров самого фотона в каждой точке траектории движения.
  2. С точки зрения фотона, траектория движения в пространстве свернуто до размеров самого фотона в каждой точке пространства фотона.

Рассмотрим какие выводы следуют из всего что мы узнали:


  1. Линия текущего времени фотона пересекает линию нашего времени под углом 45°, в следствии которого наше измерение времени для фотона является нелокальным пространственным измерением. Это значит, что если бы мы могли перемещаться в пространстве фотона, то мы бы перемещались от прошлого к будущему или от будущего к прошлому, но эта история была бы составлена из разных точек нашего пространства.
  2. Пространство наблюдателя и пространство фотона непосредственно не взаимодействуют, их связывает движение фотона. При отсутствии движения отсутствуют угловые расхождения в линии текущего времени, и оба пространства сливаются в одну.
  3. Фотон существует в одномерном пространственном измерении, в следствии которого движение фотона наблюдается только в пространственно-временном измерении наблюдателя.
  4. В одномерном пространстве фотона не существует движения, в следствии чего фотон заполняет свое пространство от начальной до конечной точки, в пересечении с нашим простраством дающее начальные и конечные координаты фотона. Данное определение говорит, что в своём пространстве фотон выглядит как вытянутая струна.
  5. Каждая точка пространства фотона содержит проекцию самого фотона во времени и в пространстве. Имеется ввиду, что фотон существует в каждой точке этой струны, представляя разные проекции фотона во времени и в пространстве.
  6. В каждой точке пространства фотона сжата полная траектория его движения в нашем пространстве.
  7. В каждой точке пространства наблюдателя (где может пребывать фотон) сжата полная история и траектория самого фотона. Данный вывод следует из первого и пятого пункта.

Пространство фотона


Давайте попробуем разобраться что из себя представляет пространство фотона. Признаюсь, трудно представить что такое пространство фотона. Разум сцепляется за привычное и пытается провести аналогию с нашим миром. А это приводит к ошибочным выводам. Чтобы представить другое измерение нужно отбросить привычные представления и начать думать по другому.

Итак. Представьте себе лупу, собирающее в фокусе всю картину нашего пространства. Допустим, что мы взяли длинную ленту и расположили фокус лупы на этой ленте. Это есть одна точка в пространстве фотона. Теперь немного передвинем лупу параллельно нашей ленте. Точка фокуса также передвинется по ленте. Это уже другая точка в пространстве фотона. Но чем отличаются эти две точки? В каждой точке есть панорама всего пространства, но проекция выполнена из другой точки нашего пространства. К тому же, пока мы передвигали лупу успело пройти какое то время. Получается, что пространство фотона в чем то похоже на кинопленку, снятую с движущегося автомобиля. Но есть некоторые отличия. Пространство фотона имеет только длину и не имеет ширину, поэтому там фиксируется только одно измерение нашего пространства — от начальной до конечной траектории фотона. Так как в каждой точке записана проекция нашего пространства, то в каждой из них имеется наблюдатель! Да да, ведь в каждой точке фиксируются одновременные события с точки зрения самого фотона. И раз уж начальные и конечные траектории фотона расположены в одной линии времени — это одновременные события для фотона, которые затрагивают его в разных точках своего пространства. В этом основное отличие от аналогии с кинопленкой. В каждой точке пространства фотона получается одинаковая картина из разных точек обзора, и отражающая разные моменты времени.

Что происходит, когда фотон движется? Пробегает волна по всей цепочки пространства фотона, когда пересекается с нашим пространством. Волна затухает когда сталкивается с препятствием и передает ему свою энергию. Но почему возникает волновая интерференция пока не совсем ясно. Возможно когда волна сталкивается с барьером, соответствующей длине своей волны, он не может преодолеть барьер и не может передать свою энергию. Если предположить, что в каждой точке пространства содержатся фотоны в состоянии покоя, то волна может передать свою энергию соседним фотонам, которые возбуждаясь подхватывают волну и обходят барьер с двух сторон. По крайней мере такое явление может объяснить математические парадоксы, в которых говорится, что фотон проходит через обе щели и не проходит ни через одну, он так же проходит через одну и проходит через другую. Но правильнее будет предположить, что соседние фотоны не находятся в состоянии покоя, а флуктуируют в своих интервалах времени. А волна нашего времени сталкиваясь с ними заставляет их флуктуировать вне своего времени. Ведь на Земле не существует точки пространства, где никогда не было и никогда не будет фотона. Но в чем же роль наблюдателя пока не совсем ясно.

А теперь посмотрим как выглядит фотон в нашем мире. С точки зрения наблюдателя пространство фотона свернуто в размеры самого фотона. По сути это самое свернутое пространство и является самим фотоном, отдаленно напоминающую струну. Струна построенная из симметричных проекций самого себя из разных точек пространства и времени. Соответственно фотон содержит в себе всю информацию о самом себе. В любой точке нашего пространства он “знает” весь путь, и все события прошлого и будущего, касающегося самого фотона. Я считаю, что фотон безусловно может предсказывать свое будущее, нужно только поставить правильный эксперимент.

Выводы


1. Остается масса вопросов, ответы на которых трудно получить без проведения экспериментов. Не смотря на то, что подобные эксперименты с двумя щелями проводились много раз, и с различными модификациями, в интернете очень трудно найти об этом информацию. Даже если удается что-то найти, нигде не приводятся вразумительных объяснений сути происходящего и анализа результатов эксперимента. Большинство описаний не содержит никаких выводов и сводится к тому что, «есть такой парадокс и никто не может его объяснить» или «если вам кажется что вы что то поняли, значит вы ничего не поняли» и т. д. А между тем я считаю, что это перспективное направление исследования.

2. Какую информацию можно передавать из будущего в настоящее? Очевидно, что мы можем передать два возможных значения, когда мы будем или не будем наблюдать за холостыми фотонами. Соответственно, в текущем времени мы будем наблюдать волновую интерференцию или скопление частиц из двух полос. Имея два возможных значение можно использовать бинарное кодирование информации и передавать любую информацию из будущего. Для этого потребуется должным образом автоматизировать этот процесс, с использованием большого количества квантовых ячеек памяти. В этом случае мы сможем получать тексты, фотографии, аудио и видео всего, что нас ожидает в будущем. Также можно будет получать передовые разработки в области программных продуктов и возможно даже телепортировать человека, если заранее отправят инструкцию, как построить телепорт.

3. Можно заметить, что достоверность получаемой информации относиться только к самим фотонам. Из будущего может быть отправлена заведомо ложная информация, ведущая нас в заблуждение. Например, если подбросили монетку, и упала решка, но мы отправили информацию, что упал орел, то мы сами вводим себя в заблуждение. Достоверно можно утверждать только то, что отправленная и полученная информация не противоречат друг другу. Но если мы решим ввести себя в заблуждение, то думаю, со временем сможем узнать, почему мы решили так поступить.
Кроме этого, мы не можем точно определить из какого времени получена информация. Например, если мы хотим узнать что произойдет через 10 лет, то нет гарантии того, что мы отправили ответ гораздо раньше. Т.е. можно сфальсифицировать время отправки данных. Думаю для решения этой проблемы может помощь криптографию с открытыми и закрытыми ключами. Для этого потребуется независимый сервер, занимающийся шифрованием и расшифрованием данных, и хранящий в себе пары открытых-закрытых ключей, сформированных на каждый день. Сервер может по запросу шифровать и расшифровать наши данные. Но пока у нас не будет доступа к ключам, мы не сможем сфальсифицировать время отправки и получения данных.

4. Рассматривать результаты экспериментов только с точки зрения теории относительно было бы не совсем правильным. Хотя бы в силу того, что СТО имеет сильную предопределенность будущего. Не приятно думать, что всё предопределено судьбой, хочется верить, что у каждого из нас есть выбор. А если есть выбор, значит должны быть альтернативные ветки реальности. Но что будет, если мы решим действовать по другому, вопреки тому, что отображается на экране? Возникнет новая петля, где мы тоже решим действовать по другому, и это приведет к возникновению бесконечного количества новых петель с противоположными решениям? Но если есть бесконечное количество петель, то мы изначально должны были видеть на экране смесь интерференций и двух полос. А значит, мы изначально не могли бы определиться с противоположным выбором, что снова приводит нас к парадоксу… Я склоняюсь к мысли, что если существуют альтернативные реальности, то на экране будет отображаться только один вариант из двух возможных, не зависимо от того, сделаем мы такой выбор или нет. Если мы сделаем другой выбор, мы создадим новую ветку, где изначально на экране будет показан уже другой вариант из двух возможных. Возможность сделать другой выбор будет означать о существовании альтернативной реальности. Мы ведь не рассматривали подробно распространение волн в пространстве фотона. Вполне возможно, что волны могут взаимодействовать с соседними фотонами, рождая новые волны, а с ними альтернативные реальности. В таком случае, волны времени пересекаясь между собой могут порождать волны вероятности. А фотоны, считающиеся переносчиком взаимодействия, могут являться полем, для распространения этого взаимодействия. Такие выводы могут вызвать массу дискуссий, поэтому я не берусь утверждать, а просто предполагаю.

5. Существует вероятность того, что как только экспериментальная установка будет включена, будущее окажется предопределенным. Возникает такой парадокс, что установка сама предопределяет будущее. Сможем ли мы разорвать это кольцо предопределенность, ведь у каждого есть свобода выбора? Или же наша «свобода выбора» будет подчинена хитрым алгоритмам предопределенности, и все наши попытки что то изменить, в конце концов сложатся в цепь событий, которые приведут нас к данной предопределенности? Например, если мы знаем номер выигрышной лотереи, то у нас есть шанс найти этот билет и получить выигрыш. Но если мы также знаем имя победителя, то мы уже не сможем ничего изменить. Может даже кто то другой должен был выиграть лотерею, но мы определили имя победителя и создали цепь событий, которая привела к тому, что предсказанный человек выиграет эту лотерею. Трудно ответить на эти вопросы без проведения экспериментальных опытов. Но если такое имеет место, то единственная возможность избежать предопределенности видеться в том, чтобы не пользоваться этой установкой и не заглядывать в будущее.

Записывая эти выводы, мне вспоминаются события фильма «Час расплаты». Поражает то, насколько точно совпадают детали фильма с нашими расчетами и выводами. Ведь мы не стремились получит именно такие результаты, а просто хотели разобраться с происходящим и следовали формулам теории относительности Эйнштейна. И всё же, если есть такой уровень совпадения, то видимо, мы не одиноки в своих расчетах. Возможно, подобные выводы уже были сделаны десятки лет назад…
Поделиться с друзьями
-->

Комментарии (153)


  1. Grief
    09.11.2016 22:32
    -4

    Далеко не факт, что эффекты, которые работают для фотона, работают для остальных частиц, из которых состоит материя. Я не слышал об экспериментах и опытах по телепортации или "перемещении во времени" частиц, отличных от фотонов, и вполне возможно, что вселенная сделана так, что для физических объектов вся эта магия недоступна.


    Ну и в порядке бреда, у меня есть предположение, что все эти эффекты как-то связаны с сознанием. Например, может быть, есть какое-то вещество в мозге, в каких-то микроскопических количествах, которое позволяет связать сигнал от нейрона с самим собой в будущем или в "параллельной вселенной" и этот эффект ощущается как сознание — ты думаешь о шершавой поверхности, ты коснешься ее рукой, а сигнал из будущего вернется в настоящий момент и ты осознаешь шершавость поверхности. Или ты пытаешься что-то придумать и ты будешь хаотично перебирать все возможные варианты и в одной из альтернативных вселенных у тебя таки что-то получится как у тысячи обезьян за печатными машинками, и сигнал оттуда опять же попадет в текущий момент и тебя озарит.


    Может даже, что это вещество проявляет это свое волшебное свойство связывать сигнал из нейрона с самим собой в другом времени только пока его не наблюдают, что делает невозможным искусственное воспроизведение сознания человеком в принципе.


    Не относитесь слишком серьезно только :)


    1. bull1251
      09.11.2016 23:17

      Я не слышал об экспериментах и опытах по телепортации или «перемещении во времени» частиц, отличных от фотонов, и вполне возможно, что вселенная сделана так, что для физических объектов вся эта магия недоступна.

      Я не имел ввиду о перемещение частиц. Под телепортацией я имел ввиду передачу полной информации об объекте. Из чего и как построен этот объект.


      1. Grief
        09.11.2016 23:27

        Ну, есть телепортация фантастическая, дискретное, а не непрерывное перемещение объекта в пространстве, есть квантовая — запутывание пары частиц, приводящее к тому, что они получают одинаковое квантовое состояние. Передача информации телепортацией не является. Плюс, я так и не понял, как предполагается передавать полезную информацию из будущего.


        1. alexeykuzmin0
          14.11.2016 11:11
          +1

          Квантовая телепортация — это не совсем

          запутывание пары частиц, приводящее к тому, что они получают одинаковое квантовое состояние.
          При квантовой телепортации разрушается запутанность между парой частиц, первая частица получает новое случайное состояние, а вторая (после передачи информации традиционным способом и воздействия на частицу) приобретает то же самое квантовое состояние, что было у первой. Собственно, существенная особенность этого процесса — невозможность копирования квантового сотояния.


  1. Shifty_Fox
    10.11.2016 00:40
    +2

    Можно попроще разжевать эксперимент с фотонами, полупрозрачным зеркалом и интерференцией? Это гипотеза, или реальная установка, демонстрирующая знание фотона о будущем?


    1. bull1251
      10.11.2016 02:00
      +1

      Можно попроще разжевать эксперимент с фотонами, полупрозрачным зеркалом и интерференцией?
      Тоже думал об этом и постарался упростить, изначально описывалось сложнее, постараюсь в ближайшее время описать по проще. Думаю также описание пространства фотона нужно будет упростить.
      Это гипотеза, или реальная установка, демонстрирующая знание фотона о будущем?
      Эти результаты подтверждены экспериментом. Проводилось множество серий экспериментов с различными модификациями. Более подробно можно прочитать например здесь http://studopedia.org/6-73200.html


      1. Shifty_Fox
        10.11.2016 14:43
        +1

        Ок, здесь важно понимать вот что.
        Фотон может быть промаркирован в левой или правой щели. Пусть так, но это никак не отразилось на его интерференции. Грубо говоря, это все еще фотон, который пролетел через обе щели, и 50% его помечено правым, а 50% левым. Далее фотон либо натыкается на стиратель бирки, либо нет. Если нет — он врезается в экран, обладая маркером спина, и волновая функция схлопывается, однозначно выбирая лево или право. Даже будь поверхность под углом — во время первой столкновении волны с поверхностью бросается кубик — здесь\не здесь, и по мере покрытия ее поверхностью, рано или поздно произойдет схлопывание.
        Вторая ситуация, когда мы попадаем на стиратель бирки, и здесь все возвращается на начало — фотон просто интерферирует сам с собой как волна, поскольку волна при столкновении с детектором не промаркирована, ее функция не схлопывается.
        Нет никакой передачи информации в будущем, есть принятие решения о схлопывании волны при ударе этой волны о что-то.

        На самом деле чтобы понять, схлопывается ли вся волна мгновенно в момент первого столкновения, нужно ставить стиратель бирки только в левую или только в правую часть. Тогда, если стерли слева, а волна справа попала позже, и при том схлопнулась, несмотря на то, что сначала в детектор ударилась волна слева, без маркера — это покажет некий перенос информации выше скорости света. Но это надо поставить эксперимент.


        1. bull1251
          10.11.2016 18:21

          Следуя вашей логики в моем примере получиться, что если детектор холостых фотонов включен, то две полосы на экране мы сможем увидеть только после того как холостой фотон достигнет детектора, а если посмотрим на экран до этого момента, то увидим интерференцию, потому как холостой фотон еще на подлете к детектору? Но раз мы заранее смотрели на экран, то стели из нашей реальности холостой фотон, и потому он никогда не достигнет детектора?


          1. Shifty_Fox
            10.11.2016 22:54

            Если мы посмотрим до того, как фотон долетел, мы еще ничего не увидим. По моей логике, фотон — волна, которая решает, схлопнуться ли в частицу — именно в тот момент — когда она соударяется с поверхностью. Свое мнение волна строит относительно того — был ли промаркирован спин фотона или не был. То есть, на том этапе, где как нам кажется, мы маркируем какой-то целостный фотон — на самом деле мы маркируем волну, а вот когда волна бьется — вот тогда маркер и проявляется. Причем без маркера волна остается волной, т.е. интерферирует.


          1. ilansk
            11.11.2016 17:52

            Ну как дети, честно слово! Получать информацию из будущего собрались обманув фотон )))
            Это напоминает вытягивание себя за волосы из болота.


    1. win32asm
      10.11.2016 02:57
      +1

      https://en.wikipedia.org/wiki/Delayed_choice_quantum_eraser
      сорри, русского варианта статьи (пока) нет

      Про возможность «изменения прошлого» говорится:
      … retrocausality is not necessary to explain the phenomenon of delayed choice. Note in particular that an interference pattern may only be pulled out for observation after the idlers have been detected (i.e., at D1 or D2).
      … инверсия причины и следствия не нужна, чтобы объяснить феномен отложенного выбора. Заметьте, что наблюдать интерференционную картину можно только _после_ получения данных об «отброшенном» фотоне.


  1. VaalKIA
    10.11.2016 01:54
    +7

    «теоретическое обоснование о возможности преодоления времени между прошлым и будущим. Чем занимался в течении последних дней» Дней, Карл! Простите, не удержался, пошёл читать статью полностью, возможно эти шутки со временем, результат каких-то практических испытаний.


    1. GoldJee
      10.11.2016 15:37

      У меня тоже после прочтения этой фразы сложилось (возможно) неверное впечатление, что это умозрительные рассуждения на досуге человека, уставшего от ведения «однообразной жизни занимаясь софтом, железом, и отвечая на однотипные вопросы пользователей».
      Хотя по тексту статьи видно, что человек более-менее разбирается в предмете, о котором рассуждает.

      Просьба дать ссылку на исследование, подтверждающее результаты мысленного эксперимента из параграфа «Замедление времени». У меня не всплывают в памяти такие научные работы. Более того, вытягивание света в линию, параллельную движущимся зеркалам, кажется мне совсем неочевидным. Также для меня неочевиден следующий из этого вывод, что «фотон не имеет ни прошлого, ни будущего».


      1. amphasis
        10.11.2016 16:11

        Не могу ничего сказать о «замедлении времени». Но вывод о том, что «фотон не имеет ни прошлого, ни будущего» делается из общей теории относительности. Грубо говоря, поскольку фотон перемещается со скоростью света, в его системе отсчета время не идет, соответственно в своей системе отсчета он излучается, проходит весь свой путь и поглощается одновременно, в один момент времени.


        1. GoldJee
          10.11.2016 16:46

          Этот вывод можно сделать и из специальной теории относительности, вспомнив концепцию пространства Минковского, на которую автор неявно ссылается по ходу статьи.
          Мое сомнение заключается в том, что этот вывод подается как следствие мысленного эксперимента с двумя зеркалами. Но для меня такая причинно-следственная связь неочевидна.

          Будет здорово, если автор прокомментирует этот параграф, дав ссылку и пояснив свои логические заключения.


          1. bull1251
            11.11.2016 18:08

            Эти выводы являются следствием специальной теории относительности, сформулированные самим Эйнштейном. Я лишь привёл примеры для наглядности. Проверить очевидность этих выводов можно с помощью формул СТО и математических расчетов.


  1. VaalKIA
    10.11.2016 02:38

    «На самом деле, нам даже не нужно знать полную цепочку событий. Необходимо достоверно узнать сбудется или нет одно конкретное событие в будущем (которое будет объектом исследования). Если сбудется, то значит, цепь событий привели к тому, чтобы это событие сбылось»
    Как бы — нет, цепь событий — оно сбылось, это — бред. Вот если цепь событий не приводит, а оно сбылось, вот тогда — да. Катишь шарик, а тут бах — его микрометиорит сбил, хотя цепь событий однозначно предсказывала что он и дальше будет катиться, вот это как раз и получиться, знать события, всё остальное — туфта.


  1. igorch96
    10.11.2016 08:06
    +2

    Вопрос без стёба: а что есть время? Насколько мне видится, время — это искусственная величина, придуманная человеком для измерения тех или иных процессов, происходящих в окружающем его мире. Или я неправ? А если неправ, то в чём?


    1. mwizard
      10.11.2016 11:32

      Если бы было так, то не было бы релятивистского замедления времени, а оно подтверждено экспериментально.


      1. Shifty_Fox
        10.11.2016 14:09
        +2

        Ну так оно релятивисткое замедление движения группы частиц на квантовом\микро уровне, что выражается в ощущении замедлении времени, и время здесь снова искусственно введенная величина.


        1. Stalker_RED
          10.11.2016 23:57

          Почему только на микро? На спутниках GPS регулярно корректируют часы, т.к. спутники движутся быстрее чем поверхность земли. Вполне себе макро объект.


          1. Shifty_Fox
            11.11.2016 01:55

            Да, все так.
            Я говорил о том, что нужно изменить точку восприятия. Замедляется «течение» объекта, его скорость «исполнения». Причинно-следственные связи на месте, времени нет, есть только понятие скорости изменения состояния объекта.
            Что есть часы на спутниках GPS? Это физические объекты, которые меняют состояние, и через их состояние мы условно меряем время. Объекты оказались в гравитационном поле\движутся с ускорением — их скорость смены состояний меняется, и мы вынуждены корректировать их, чтобы их именно «условное состояние», положение стрелки часов — совпадало с земным.


          1. saboteur_kiev
            11.11.2016 06:31

            Они не «движутся быстрее». Чуть выше очень правильно спросили — что такое время.

            Во вселенной происходят процессы взаимодействия различных объектов.

            Распространение процессов ограничено причинно-следственной цепочкой, поэтому оно не одновременное а последовательное При этом не единичное, а множественное.

            Возможность измерить эту последовательность мы называем время.
            Но оно просто неоднородно, в том смысле, что последовательность взаимодействий между разными объектами может происходить «быстрее» или «медленнее» из-за различных условий, в том числе и количестве объектов на единицу пространства.
            То, что часы в условиях невесомости идут иначе, чем в условиях земли (с давлением и атмосферой, плюс гравитационные искажения), никак не означает, что они «путешествуют во времени».


      1. Ckpyt
        11.11.2016 01:46

        дык это… увеличивается время, необходимое для волны, чтобы перебежать от одной частицы к другой -> замедляется время. Но это эффект замедления времени, а не замедление времени.


    1. GoldJee
      10.11.2016 15:48

      С точки зрения физика это параметр в уравнениях движения. Координата в четырехмерном пространстве, если хотите.


    1. pr_Black
      10.11.2016 22:20

      Что есть время (имхо), рассматривается в зависимости от контекста. Можно рассматривать время как функцию от расширяющего пространства (от большого взрыва), можно рассматривать как 4-ю координату пространства (траектории пешехода и автобуса пересеклись в точке по совпадающим 3-м координатам, но отличались по 4-й).
      Есть еще интересная точка зрения (немного из философии) если есть нечто (фотон) чья скорость постоянна относительно любой системы отсчета (и это максимально достижимая скорость в нашей реальности), то должно существовать нечто чья скорость постоянна и равна 0 относительно любой системы отсчета. Мне кажется что это нечто с нулевой (и возможно как раз нулевая скорость и есть гиперсветовая) связано с гравитацией поскольку должно существовать одновременно везде. Если фотон существует одновременно во всем своем времени, то нечто должно существовать одновременно во всем своем пространстве.
      Еще раз это все ИМХО.
      С уважением к автору.


    1. Zakhar0v
      10.11.2016 22:22

      Попробую объяснить свою точку зрения, если ошибаюсь, пусть специалисты поправят, так как сам хочу знать истину.
      Есть секунда — интервал времени, равный 9 192 631 770 периодам излучения, соответствующего переходу между двумя сверхтонкими уровнями основного состояния атома цезия-133, находящегося в покое при 0 К. (С википедии).
      И как я понимаю, это величина непостоянна в различных условиях (например на земле и в условном чайнике между орбитами, особенно если этот чайник, допустим, ускоряется).
      То есть, время — скорость процесса, относительно которой можно судить о скорости других процессов в этих же условиях.
      А есть философское понятие времени, то есть процесс от прошлого к будущему, в данной точке пространства-времени равный физической величине.
      То есть, философское понятие — это что то вроде наблюдателя, находящегося «за пределами ВСЕГО», скажем так, внешний — и поэтому, мы никак не можем гарантировать(раз сами находимся внутри), что «вчера», физическая и филосовская величина секунды были тоже равны.
      И если теоретически поставить себя как внешнего наблюдателя, то процесс «улетания»от земли звездолета, на близкой к световой скорости, будет еще отмечен тем, что на земле, электроны в атомах, грубо говоря, облетели ядро тысячу раз, а на звездолете всего лишь сто.
      То есть, все парадоксы заключаются в путанице между философским и физическим смыслом.


      1. ilansk
        11.11.2016 18:06

        С физической точки зрения время: это когда физик берет относительно себя какие-то циклические изменения за эталон, потому что они ему кажутся независимыми и сравниваем их с другими изменениями. А потом подсчитывает сколько изменений уложилось в другие изменения. Так за 1 ден происходит 24 часа, а 1 с свет проходит 300000000 км и т.д. выбор циклических изменений меняется взятых за эталон времени то же меняется: раньше секунда привязана была к вращению солнца, а теперь к Цезию-133. Мудрец был прав: в этом мире постоянны только перемены!


        1. Zakhar0v
          11.11.2016 18:21

          А эталона, относительно которого можно измерять при любых условиях, у нас, как я понимаю, нет и не предвидится, или чтото есть?

          Зануда mode
          Когда учился в школе, за слова «это когда», в определениях, можно было сразу получить два.


    1. Wizard_of_light
      10.11.2016 23:54

      Вещи вокруг нас склонны изменяться, мы называем этот процесс временем. Но с материалистической позиции величина как физическое свойство существует независимо. Человек только вычленяет её из внешнего мира (иногда достаточно произвольно), придумывает для неё термин и выбирает эталон.


  1. nvksv
    10.11.2016 10:38
    +1

    Квантовая механика — вообще штука веселая. И с ОТО (включая СТО) она не дружит. Сможете успешно подружить (а именно это Вы и пытаетесь сделать) — вэлком то Стокгольм.

    Приведу еще несколько примеров таких временных парадоксов квантовой механики. Во-первых, простейший двухщелевой опыт: максимумы интерференции возникают не только строго посередине между щелями, но и сбоку, откуда до щелей разное расстояние. Поскольку частица (или фотон) интерферирует не с другими, а только сама с собою, получается, что интерференция в данной точке возникает между двумя путями с разными подлетными временами.

    Во-вторых, виртуальные частицы. Вакуум наполнен непрерывно рождающимися и аннигилирующими виртуальными частицами (включая фотоны). Причем они рождаются в парах и после рождения проходят разные пути. Виртуальные фотоны — так вообще макрорасстояния проходят, образуя электромагнитные поля. А на это требуется прорва времени. И тем не менее, суммарный баланс и законы сохранения как-то соблюдаются.

    Как вариант если не решения таких временных проблем, то хотя бы их откладывания до лучших времен, можно принять такой «антропный» принцип: мы видим только те варианты Вселенной (Мультивселенной), в которых не происходит нарушения законов сохранения (квантмех) и причинности (ОТО). А это на автомате запрещает все машины времени, увы: даже если что-то передается в прошлое, воспользоваться этим у нас не получится.

    P.S. Уилер и Фейнман на полном серьезе строили теорию, в которой информация о каждом событии уносилась со скоростью света в прошлое (т.е. сейчас я уронил чашку, а на Проксиму Центавра информация об этом пришла еще 4 года назад). Но что-то у них все-таки не срослось.


    1. kazaff
      10.11.2016 15:31

      Хм, прочитав ваш пост, мне теперь кажется, что медное ведро из вчерашнего поста просто отталкивается от виртуальных частиц, пока они существуют.


      1. nvksv
        10.11.2016 18:35

        Идея хорошая, да законы сохранения мешают. Виртуальные частицы рождаются всегда парами противоположных, отталкивание от одних всегда будет компенсироваться притяжением к другим, а в сумме всегда выйдет нуль. Есть тонкие эффекты, где свойства физического вакуума проявляются (тот же самый эффект Каземира, всякие перенормировки и т.д.), но в целом Вселенная за законами сохранения следит очень строго.


    1. CaptainFlint
      10.11.2016 15:48
      +1

      Квантовая механика — вообще штука веселая. И с ОТО (включая СТО) она не дружит.
      Вроде же, с СТО нормально дружит. Начиная с уравнения Дирака и заканчивая квантовой теорией поля, которая целиком релятивистская.


  1. GerrAlt
    10.11.2016 10:58
    +1

    А не могли бы пояснить один момент в эксперименте с двумя зеркалами и лучом света:

    если вы двигаете зеркала в то время как луч отражается между ними, не значит ли это что луч будет каждый раз попадать не в то место зеркала, в которое он попал при прошлом отражении, и таким образом когда вы начнете двигать зеркала со скоростью света то вы в какой-то момент просто выдерните зеркало из под луча?

    Т.е. что заставляет луч двигаться вместе с зеркалами?


    1. tretyakovpe
      10.11.2016 15:31

      скорость света же


      1. GerrAlt
        10.11.2016 16:15

        Возьмем два зеркала и расположим их противоположно одну над другой. Допустим, что луч света будет многократно отражаться между этими двумя зеркалами. Движение луча света будет происходить по вертикальной оси, при каждом отражении отмеряя время как метроном. Теперь начнем двигать наши зеркала по горизонтальной оси


        луч света двигается со скоростью света от зеркала к зеркалу, зеркала двигаются со скоростью света ортогонально направлению движения луча (иначе не понятно как должна бы получаться картинка похожая на пружину)

        Из описания эксперимента я так понял что луч должен в процессе эксперимента набирать скорость в том же направлении что и зеркала, иначе как мне кажется мы таки выдернем из под него зеркала и луч просто улетит, но за счет чего луч будет приобретать эту скорость?

        Т.е. за счет чего луч света приобретает скорость позволяющую ему менять свое положение в пространстве относительно зеркала (любого из двух) только по 1 оси?


        1. tretyakovpe
          10.11.2016 16:18

          если не двигать зеркало, луч улетит вперёд и в сторону последнего отражения.
          если зеркала двигаются со скоростью света, тогда луч всё время находится между ними.
          я так понял.
          про то, откуда берется энергия на бесконечное отражение, думаю не стоит задумываться


          1. GerrAlt
            10.11.2016 16:32

            не понял вашу мысль

            по условиям зеркала одно напротив другого, т.е. луч заперт между ними (если я правильно понимаю то в начале эксперимента угол падения луча 90 градусов), коэффициент отражения мы считаем =1, потерями при прохождении через среду между зеркалами пренебрегаем

            в таком виде установка стабильна, луч летает между зеркалами и никуда не девается

            далее мы начинаем двигать зеркала ортогонально направлению движения луча, и вот тут возникает вопрос: за счет чего у луча должна увеличиваться скорость в том же направлении что и у зеркал?


            1. tretyakovpe
              10.11.2016 16:35

              не вижу в условии угла падения 90 градусов.
              на картинке в посте вижу «пилу» отраженного луча. вот эта «пила», продлённая в бесконечность и становится лучом видимо


              1. GerrAlt
                10.11.2016 16:47
                +1

                Допустим, что луч света будет многократно отражаться между этими двумя зеркалами. Движение луча света будет происходить по вертикальной оси, при каждом отражении отмеряя время как метроном.


                судя по упоминанию вертикальной оси предполагается ортогональность направления движения луча и плоскости зеркал


    1. bull1251
      10.11.2016 17:33
      -1

      Совершенно верно. Тут нужно рассматривать так, что луч света двигается вместе с зеркалами, значит они покоятся относительно друг друга. Например Земля двигается с огромной скоростью относительно млечного пути, но луч света выпущенный на Земле не меняет своей траектории относительно зеркал, так же расположенных на Земле. Хотя относительно млечного пути наш луч двигается диагонально, и проходит большее расстояние чем на Земле, значит время на Земле течет чуть медленней.

      Выдернуть зеркало из под луча можно, если они двигаются неравномерно относительно друг друга. То есть когда зеркала будут укоряться быстрее относительно луча. Это говорит о том, что невозможно ускорить движение зеркал, сохранив при этом равномерность движения с лучом. Но данный пример приводилось в тексте чисто гипотетически, для простоты понимания сути происходящего.


    1. nvksv
      10.11.2016 18:38

      Фотоны всегда двигаются со скоростью света. Любые зеркала (да и вообще любые тела, обладающие массой покоя) всегда двигаются со скоростью строго меньшей скорости света.


  1. UncleAndy
    10.11.2016 11:56
    +3

    «Какую информацию можно передавать из будущего в настоящее? Очевидно, что мы можем передать два возможных значения, когда мы будем или не будем наблюдать за холостыми фотонами. Соответственно, в текущем времени мы будем наблюдать волновую интерференцию или скопление частиц из двух полос.»

    Как-то долго размышлял над этой проблемой. И советовался со знающими людьми. По тому обсуждению получилось что понять получилась или нет интерференционная картина получится только ПОСЛЕ того, как будет получена информация о факте наблюдения за вторым фотоном. Я не понимаю механизм работы этого явления, но объяснение было таким.

    Я тогда думал что с помощью наблюдения за интерференционной картиной можно будет передавать информацию быстрее скорости света. Что, в общем-то, совпадает с тем, что вы описываете.


    1. tretyakovpe
      10.11.2016 15:34

      проблема с посланием из будущего в том, что мы не можем в настоящем определить, что это оно, послание.


      1. UncleAndy
        10.11.2016 15:40

        Да не совсем. Можно как сигнал использовать сам факт смены интерференционной картины на две линии и наоборот. И дальше просто передавать той-же азбукой Морзе, например. На самом деле, здесь есть какие-то более фундаментальные трудности.


        1. tretyakovpe
          10.11.2016 15:42

          фундаментальная трудность в том, что нам нужно знать, что инфа пошла. т.е. начало сессии. но мы об этом узнаем только когда доживём до этого.


          1. QDeathNick
            10.11.2016 16:45

            Ну не сложно же дожить. Вот я собираюсь нажать кнопку передачи, настраиваю приёмник, и как настроил, вдруг раз, уже пошли данные, а я только потом нажимаю кнопку передачи.

            И тут я передумал нажимать :)
            Вот в этом по-моему фундаментальная трудность.


            1. tretyakovpe
              10.11.2016 16:47

              кстати да. зачем отправлять инфу в прошлое, они там все дураки и ничего не поймут :)


            1. hogans
              11.11.2016 07:37

              просто не пойдут данные, если передумаете нажимать
              пойдут, если однозначно нажмете
              будущее предопределено, а фундаментальной трудностью как раз будет это осознать


    1. EvgK
      11.11.2016 00:08
      +1

      Объяснение было совершенно верным. Обычно в таких экспериментах (а их версий существует множество), «сигнальные» фотоны не образуют никакой картины и их расположение хаотично, и таким оно и остается до тех пор, пока мы не узнаем судьбу «холостых» фотонов. Дальше мы выбираем из этой хаотической картины сигнальные фотоны, соответствующие некоторым «холостым» (например, холостых зафиксированных одним из детекторов в нашей схеме) и видим, что действительно, эти соответствующие фотоны образуют интерфереционную картину (или наоборот — не образуют, в зависимости от деталей эксперимента). Поскольку до получения информации о состоянии холостых фотонов (обычным путем, не выше скорости света) мы видим только случайное распределение сигнальных — получить таким образом информацию «из будущего» (или «выше скорости света», что по сути одно и то же) мы не можем.


      1. UncleAndy
        11.11.2016 00:42

        Спасибо за пояснение. Именно что-то такое там и было.


      1. XoJIoD
        11.11.2016 09:49

        А не могли бы вы также написать, что мешает подмножеству зафиксированных фотонов быть равным множеству испущенных? Т.е. почему мы можем получить информацию только о некоторых из холостых фотонов, а не о всех?


        1. EvgK
          11.11.2016 11:42

          Мы можем получить информацию о всех холостых фотонах, и обычно в этих экспериментах так и происходит (поскольку если мы не измерили холостой фотон, то никаких выводов о сигнальном сделать не можем). Но в таких экспериментах холостые фотоны делятся на группы. Например, если фотон зафиксирован детектором А (условно) — мы однозначно знаем, которой путь выбрал сигнальный фотон. Соответственно, если взять все сигнальные фотоны, соответствующие холостым из детектора А — мы не увидим никакой интерференции. Также у нас есть детектор Б, куда попадают все остальные холостые фотоны (то есть мы засекаем все холостые фотоны, либо в А либо в Б). Если фотон попал в детектор Б — мы не можем определить, какой путь выбрал сигнальный фотон. Соответственно, если взять все сигнальные фотоны, соответствующие холостым из Б — они дадут интерференционную картину (что и подтверждается экспериментом). Однако если просто взять все сигнальные фотоны (соответствующие А + Б) — мы наблюдаем просто случайное хаотическое распределение фотонов.


          1. XoJIoD
            11.11.2016 12:40

            Это ясно, спасибо. Но суть описанного эксперимента заключается в том, что мы откладываем решение о том, куда направить холостые фотоны. Т.е. уже после попадания сигнальных на экран мы решаем, направить ли холостые на детектор А (который даст нам однозначную информацию о пути), либо на детектор Б (или вообще вникуда). Что будет в этом случае?


            1. EvgK
              11.11.2016 22:09

              Я сам пост читал только мельком, теперь прочитал этот момент более внимательно. В посте предполагается, что после того как мы поставили два даун-конвертера, мы все еще можем получить интерференционную картину на экране, но это не так. Что бы мы ни делали с холостыми фотонами — на экране никогда не возникнет интерференции (в установке, описанной в статье). Поэтому, все что описано далее в статье, основанное на этом предположении, попросту не верно. И детектирование холостых фотонов очевидно нам ничем не поможет, поэтому так, как описано в статье этот эксперимент и не проводят. В книге, откуда автор скопировал это описание (за исключением вывода), это лишь начало описания эксперимента, далее установка расширяется другими устройствами.


              1. XoJIoD
                11.11.2016 22:44

                Спасибо вам за ответ. Правильно ли я понимаю, что если мы все же придумаем установку, в которой решение, детектировать ли холостые фотоны, отложить, то на экране мы всегда будем видеть двухщелевую (или Х-щелевую, в зависимости от опыта) картину сигнальных фотонов? Т.е. они каким-то образом «догадываются», что их путь можно отследить, и теряют свои волновые свойства?


                1. EvgK
                  11.11.2016 23:11

                  Но мы можем отложить это решение прямо в эксперименте из статьи, однако на экране никогда не будет интерференции (независимо от того, когда мы детектируем холостые, и детектируем ли вообще). Возможно вы имеете в виду не двухщелевую (интеренфереционную) картину, а наоборот? Тогда все верно.


                  1. XoJIoD
                    11.11.2016 23:16

                    Под двухщелевой картиной я подразумевал две полоски на экране (т.е. отсутствие интерференции). Вроде бы ясно, спасибо)


                    1. EvgK
                      11.11.2016 23:24

                      Все же две полоски это интерференция (взаимное усиление/ослабление), а не ее отсутствие :) Хотя конечно зависит от контекста…


                      1. XoJIoD
                        11.11.2016 23:27

                        Позвольте, но ведь в классическом двухщелевом опыте (с детекторами частиц в каждой из щелей) мы и будем видеть ровно две полоски?


                        1. EvgK
                          11.11.2016 23:34

                          Да, позже мне пришло в голову что вы имеете в виду двухщелевой опыт. Просто интерференционная картина тоже состоит из полос, что меня и смутило :)


                1. EvgK
                  11.11.2016 23:18

                  Вообще, вместо неверно интерпретированного эксперимента из статьи, рекомендую почитать про другой. Он называется «квантовый ластик с отложенным выбором». Смысл его такой: сигнальные фотоны точно так же как в экперименте из статьи идут на экран и образуют хаотическую картину. Холостые же подвергаются определенным манипуляциям и попадают (гораздо позже, хоть через 100 лет), в один из двух детекторов. Если они попали в детектор А, то невозможно определить, по какому пути они шли. Если взять все сигнальные фотоны, соответствующие холостым из А, то мы увидим полосы (интерференцию), среди хаоса изначальной картины. Слово ластик в названии отражает тот факт, что путем манипуляций информация о том, какой путь выбрал фотон была «стерта», и вот в этом случае мы наконец видим интерференцию (но только для части фотонов, все вместе они по прежнему выглядят случайно).


                  1. XoJIoD
                    11.11.2016 23:22

                    Я читал про данный эксперимент. Оставался невыясненным только факт, теряют ли волновые свойства фотоны, которые теоретически могут быть отслежены. В ластике мы специально им «говорим», что не могут, поэтому интерференция снова проявляется


                    1. EvgK
                      11.11.2016 23:30

                      В ластике мы детектируем две группы фотонов. Для одной группы интерференция есть, а для другой нет. Поэтому в зависимости от манипуляций они либо «теряют» волновые свойства, либо нет (что конечно само по себе удивительно, но не позволяет передавать информацию быстрее скорости света или в прошлое).


                      1. XoJIoD
                        11.11.2016 23:32
                        +1

                        Ну, раз уж речь зашла, то позвольте еще вопрос) В эксперименте с ластиком, если мы удалим маркеры со всех холостых фотонов, но после того, как сигнальные дошли до экрана, эти сигнальные ведь также будут показывать отсутствие интерференции?


                        1. EvgK
                          11.11.2016 23:54

                          Я не физик, поэтому могу ошибаться. Но насколько я понимаю, в эксперименте с ластиком у нас 4 детектора: D1, D2, D3, D4. Если фотон попал в D1 или D2 — информация о пути «стерта». Если мы удалим маркеры со всех фотонов, они все попадут в D1 или D2. Но важный момент в том, что мы получим ДВЕ интерференционные картины (для фотонов из D1 и D2 соответственно), причем пики одной совпадают с впадинами другой. Соответственно эти две картины в сумме дают обычное хаотическое распределение (которое мы и видим на экране). Вот тут: https://en.wikipedia.org/wiki/Delayed_choice_quantum_eraser есть картинка, показывающая как выглядят результаты (Figure 4). Видно, что если сложить R01 и R02 — получим хаотический набор точек.


      1. lakroft
        11.11.2016 15:30

        Правильно ли я понял, что накладываются друг на друга картины от разных фотонов, и зарегистрированных и нет? И это никак нельзя обойти (генерировать единичный фотон, увеличить время детектора)?


        1. EvgK
          11.11.2016 22:14

          В таких экспериментах обычно как раз и генерируют по одному фотону, потому что сопоставление сигнального и соответвующего ему холостого ведется на основе времени (сигнальный испустили во время Х, холостой детектировали в Х + А\с (где А — длина пути до детектора). Наложение картин от разных групп фотонов обойти действительно никак нельзя — информацию о том какой фотон принадлежит какой группе мы получаем только после детектирования холостого партнера.


  1. Vlad_fox
    10.11.2016 12:02
    -3

    кхе, как человек, тоже не занимающийся ранее физикой серьезно, решил тож посвятить пару минут этому важному вопросу. он меня тоже заволновал во время бешлого чтения статьи.
    И вот что хочу сказать — все эти кванты, которые засовывают в щели — это все наверное кому-то важно и нужно, но не это главное. А что спросите вы?
    главное то, что как всякое путешествие — путешествие вовремени не простое дело и длительное.
    а значит надо запастись харчами — лучше сырокопченой колбасой, сыром, шоколадом (это если путешествуете в будущее, бо качество харчей постоянно ухудшается) или антибиотиками, батарейками для плеера и презервативами — если в прошлое, бо там с ними не очень.
    деньги лучше или баксы США или золото.
    О да фотик захватите.
    И книжку какую-нибудь интерсную про засовывание фотонов в щели, чтоб скоротать время во время путешествия.


    1. V-core
      10.11.2016 15:34

      Интересная книжка про про засовывание фотонов в щели — "Предсказывая свет" автор Тед Косматка


  1. vinograd19
    10.11.2016 13:00

    Не до конца понял идею.

    У меня есть два яблока: красное и зеленое. Красное я кладу в коробку и закрываю коробку. Через минуту я открою коробку, но уже сейчас я знаю, что я точно увижу там красное яблоко. Это тоже путешествие во времени?


    1. Sergiy
      10.11.2016 14:57

      Скорее у вас яблоко, цвет которого становится известным, как только вы на него посмотрите, а до этого оно не красное и не зеленое.


      1. vinograd19
        10.11.2016 14:58

        Это никак не отвечает на мой вопрос, однако.


        1. tretyakovpe
          10.11.2016 15:36
          -1

          что непонятно? у вас передача инфы из прошлого в будущее, а тут разговор об обратном. т.е. вы должны получить «флэшбэк» о том, как через несколько секунд будете класть яблоко.
          Может эффект дежавю это как раз они, послания из будущего?


          1. Grief
            11.11.2016 00:23

            А память — послания из прошлого. Она более «стабильна», потому что прошлое одно, а будущих много. Я об этом в самом первом сообщении написал. Хотя, скорее всего, на самом деле дежавю — просто небольшой глюк, ложное воспоминание.


            1. bull1251
              11.11.2016 00:35

              Я бы не стал утверждать что прошлое одно. Это еще не доказано.
              Вариантов прошлого может быть столь же много, сколь и будущего. С повседневной точки зрения нам конечно кажется, что у нас одно прошлое. Но с этой точки зрения будущее также должно иметь только один вариант. Ведь возможность выбора остается только возможностью, пока выбор не сделан, а мы не можем одновременно выбрать два возможных варианта, взаимоисключающих друг друга.


              1. tcapb1
                11.11.2016 14:55

                С одной стороны — да, не доказано. С другой стороны, прошлое имеет меньшую энтропию, чем будущее. То есть количество возможных микросостояний на макросостояние должно быть меньше, а значит, насколько я понимаю, вариантов прошлого должно быть меньше, чем вариантов будущего.

                А если верна многомировая концепция Эверетта, то и утверждать что будущее только одно мы не можем.


                1. bull1251
                  11.11.2016 15:05

                  Это не совсем так. Второй закон термодинамики подразумевает, что энтропия возрастает по направлению в будущее от любого заданного момента, ибо вещи скорее всего будут разупорядочиваться, так как для беспорядка существует больше возможностей, чем для порядка. Но поскольку известные законы природы рассматривают направления вперёд и назад во времени как совершенно равноправные, второй закон в действительности означает, что энтропия возрастает как в направлении будущего, так и в направлении прошлого от любого заданного момента. Это значит, что если время повернуть вспять, то движение всех объектов в пространстве может происходить в том же порядке, как при обычном движении во времени. Например, снег будет также падать на Землю, потому что гравитация не станет отталкивать снежинки.
                  image


                  1. Wizard_of_light
                    12.11.2016 14:44

                    Таки нет, по второму закону термодинамики в замкнутой системе прошлая энтропия всегда меньше или равна будущей. Просто теория процесса обычно сильно упрощается, если знак времени можно не учитывать. А если изменение энтропии системы пренебрежимо мало во время протекания в системе изучаемого процесса, то направление времени на рассматриваемый процесс влияет мало. Этим-то и злоупотребляют пользуются.


      1. Rampages
        10.11.2016 18:09

        Яблоко Шрёдингера уже какое-то получается :).
        Пример у vinograd19 не совсем удачный и не является примером путешествия во времени.


  1. Jetmanman
    10.11.2016 13:11
    -8

    Не все понял. Но возможность путешествия во времени не создает противоречий и логически возможно, другое дело какие здесь ограничения из физики. Судьба существует, я об этом писал в своей статье https://geektimes.ru/post/279780/, а то, что людям не хочется думать, что судьба есть это лишь их желания так думать. Вы сами написали почему судьба есть, но начинаете выдумывать почему ее может не быть. И вот еще что скажу, люди не понимают, что прошлое также существует как настоящее и будущее, это можно доказать из существования парадокса близнецов например, когда один близнец молоде другого и соответственно видит будущее второго близнеца. В реальности вместо близнецов есть космонавты, которые возвращаясь на землю чуть моложе остальных людей, т.е. находятся в их будущем. А в той своей статье я еще писал можно ли предсказать будущее.


    1. LexS007
      10.11.2016 16:01

      Ах вот оно что. Ну все, пошел смотреть «битву экстрасенсов».


      1. Jetmanman
        10.11.2016 19:09
        -1

        Держи в курсе.


  1. rerf2010rerf
    10.11.2016 14:15
    +4

    Сидят там чего то эти физики в сотнях институтов и университетов по всему миру, сначала изучают физику много лет, потом ещё больше научной работой занимаются, готовят себе замену, сменяются поколения, и так уже несколько сотен лет. Ставят эксперименты, накапливают данные по крупицам, годами их обрабатывают, строят теории, как-то их объясняющие, зачастую ошибаются… И до сих пор не научились во времени путешествовать, вот ведь гады.
    Ну ничего, сейчас придёт железячник с гиктаймса, отвечающий на однотипные вопросы пользователей, на днях задумавшийся о путешествиях во времени, и докажет всей это учёной братии, как надо физику делать.
    Вы это всерьёз? Вам самому то не смешно?


    1. gsaw
      10.11.2016 15:51
      -2

      Судить о человеке по тому, чем он работает еше смешнее. Эйнштейн в патентном бюро работал, Циалковский в школе учителем, Франклин был газетчиком, послом.


      1. rerf2010rerf
        10.11.2016 16:36
        +2

        Так я не о человеке, я о ситуации в целом. Что-то мне подсказывает, что все вышеупомянутые люди сделали несколько больше, чем несколько дней думали в высоком.


        1. bull1251
          11.11.2016 16:20
          +1

          Ну вообще то я закончил физико-математический факультет. И в какой то степени разбираюсь в данной области. К тому же нельзя сказать, что построил такую теорию за пару дней. В течении многих лет я задумывался над этими вопросами, с каждым разом все ближе и ближе подбираясь к сути, но не мог уловить самой основы, не мог представить в себе как выглядит пространство фотона. А после прочтения указанной статьи я с большей силой заинтересовался этими вопросами и решил постараться выстроить полную картину, собрав в едино все имеющиеся свои идеи. В процессе детального анализа я вдруг осознал как нужно смотреть, чтобы представить пространство фотона.


        1. bull1251
          11.11.2016 16:41

          Имеются ли у вас основание обвинять других в некомпетентности? Для подобных обвинений, я полагаю, вы должны лучше разбираться в данной области. В таком случае, не могли бы вы конструктивно объяснить в чем заключаются ошибки в моих суждениях?


          1. rerf2010rerf
            11.11.2016 17:11
            +1

            Да не надо разбираться в подобных вещах, чтобы увидеть, что у вас бессвязный поток слов. Ну да ладно, развлекусь немного

            Линия текущего времени фотона пересекает линию нашего времени под углом 45°...

            Вы уж определитесь, у вас для фотона времени не существует, или оно чего-то там пересекает. Это вы о световом конусе так оригинально изъясняетесь что-ли? Так он к «времени фотона» под которым, как я понимаю, вы имеете в виду собственное время, хоть и не знаете этого, не имеет ни малейшего отношения. А собственное время чего-бы то ни было не может ничего пересекать, потому что это не линия.
            Пространство наблюдателя и пространство фотона непосредственно не взаимодействуют

            Если перевести это на человеческий язык, то читать надо примерно так «система отсчёта наблюдателя не взаимодействовать с системой отсчёта фотона...» Фраза несомненно, «верная», ибо во первых с фотоном нельзя связать систему отсчёта, а во вторых системы отсчёта не могут в принципе ни с чем взаимодействовать, ибо это всего лишь «угол зрения» на систему, а не какой-то физический объект. Так что фраза напрочь лишена смысла.
            Фотон существует в одномерном пространственном измерении

            Очевидно, это бред, ибо фотон таки трёхмерный.
            в пространственно-временном измерении наблюдателя

            Что это за зверь такой я не знаю, и физика тоже. Это у вас тоже такое извращённое название СО? Короче, терминологию науки, в которой делаете великие открытия, вы не знаете. И саму науку тоже.
            В общем, мне надоело, думаю этого хватит, чтобы показать, чего стоят ваши откровения. У вас так можно любую фразу разбирать, ибо весь ваш текст — набор несогласующихся по смыслу наукообразных терминов.


            1. bull1251
              11.11.2016 17:27
              -2

              Ну в общем мне тоже надоело с вами спорить. Я конечно мог бы попытаться вам объяснить, но думаю в этом нет смысла, ибо вы всё равно будете стоять на своем. Пусть лучше другие участники рассудят кто из нас и в чем ошибается.


            1. bull1251
              12.11.2016 22:22

              Возможно я слегка негативно отнёсся к вашим комментариям. Но чтобы исключить возможность неправильного понимания основного текста я всё же решил ответить на ваши вопросы:

              Линия текущего времени фотона пересекает линию нашего времени под углом 45°
              Для фотона не существует времени. Точнее нет движения времени, а есть только текущий момент. Это как представить пространство, в котором нет никакого движения и всё застыло на своих местах. Но если есть застывшее пространство, то значит есть текущий момент в котором оно существует (иначе это пространство не могло бы существовать). Как я писал, у фотона есть своё пространство, которая выглядит как вытянутая струна. И каждая точка этой «струны» пересекают наше пространство не одновременно. Начальная точка «струны» пересекает раньше всех, последующие точки чуть позже, и так по цепочки, пока последняя точка «струны» не пересечет позже всех. Чтобы это было возможным, текущий момент времени фотона (в котором оно пребывает) должно пересекать текущее время наблюдателя под определенным углом. А чтобы определять угол пересечения, необходимо рассматривать текущий момент времени наблюдателя как линию времени. Для скорости света этот угол уже подсчитан и составляет 45 градусов.

              Пространство наблюдателя и пространство фотона непосредственно не взаимодействуют
              Вы правильно подметили, что система отсчета это не физический объект, чтобы взаимодействовать с другой системой отсчета. Но смысл этой фразы заключается в том, что пространство фотона (или струны) существует только в пределах самого фотона (или ограничено границами этой «струны»). В нашем пространстве нет такого ограничения, чтобы за пределами измеряемого объекта заканчивалось само пространство в котором измерен этот объект. Поэтому это следовало отметить, чтобы написать следующий из этого вывод — при отсутствии движения отсутствуют угловые расхождения в линии текущего времени, и оба пространства сливаются в одну.

              Фотон существует в одномерном пространственном измерении
              Это мы живем в трехмерном пространственном измерении. Но чем больше скорость движения объекта, тем больше сжимаются относительно неподвижные объекты в пространстве. При скорости света наше пространство будет сжато до размеров самого фотона по направлению его движения. Также как пространство фотона будет сжато для неподвижного наблюдателя (до таких же размеров). Но для самого фотона его пространство остается вытянутой струной, которая составлена из каждой точки в каждом моменте времени траектории его движения. Поэтому для фотона существует только одно пространственное измерение. Если для фотона могло существовать хотя бы два измерения, то мы наблюдали бы фотон как вытянутую линию, а не точку.

              в пространственно-временном измерении наблюдателя
              Для фотона существует только пространство, для наблюдателя существует также время. Поэтому я написал именно так, чтобы отличить одно от другого. Возможно правильнее было бы написать пространственно-временном континууме, но мне казалось, что это только усложнит понимание для читателя.

              Приведенные в тексте выводы следуют исходя из объединения всех описанных изменений пространства и времени (таких как смещения угла текущего момента времени, замедления времени, сжатия пространства) как для наблюдателя, так и для фотона. Поэтому требуют некоторого обдумывания, чтобы правильно их понять.


    1. Alland456
      10.11.2016 16:14
      -3

      На досуге, посмотрите фильм «Масло Лоренцо». Основан на реальных событиях.
      Вкратце, фильм о том, что поражённые и разочарованные некомпетентностью медиков, родители решают сами искать средство лечения для своего угасающего на глазах сына. Причем, оба были очень далеки от медицины. И в результате они нашли способ победить болезнь. Их сыну это уже правда не помогло, но это позволило спасти тысячи других детей, дать им возможность нормально жить, расти, развиваться. Тем, кому профессора медицины, практики, которые много много лет изучали, продвигали, практиковали медицину, так и не смогли бы помочь.


      1. rerf2010rerf
        10.11.2016 16:34
        +5

        Да да, и всё это, конечно, сидя «несколько дней» дней на диване, и почитав по теме несколько научно-популярных статеек, судя по уровню владения материалом у автора. Конечно, именно так и делаются великие научные открытия.


    1. Olorin111
      11.11.2016 15:32

      Стоило бы немного поосторожнее со столь радикальными взглядами. Так недалеко и до схоластиков средневековья. Даже железячнику с гиктаймса может прийти в голову как в корне неверная гипотеза, так и совершенно гениальная в силу тех или иных причин. В частности — свежести взгляда. На ум сходу не приходят исторические аналоги, но вспомнилась вот фраза: Таль стал чемпионом мира, не умея играть. Сейчас научился — и больше не станет. Виктор Корчной Это я не к тому, что нужно признавать любой бред, а скорее — тут нужно все хорошенько обдумать.


      1. rerf2010rerf
        11.11.2016 16:10
        +1

        В голову железячника несомненно может прийти гениальная идея, если железячник перед этим как минимум выучит Ландау-Лифшица и ещё несколько более современных книг и статей по интересующей его теме. На всё это у железячника уйдёт лет 10 упорного труда, а не несколько дней сидения на диване. А на том уровне, на котором рассуждает наш железячник, открывать нечего. В частности, задачки по СТО, о которых он разглагольствует, решаются на первом курсе физфака. По квантовой механике — на втором или третьем. Сотнями. Этой физике уже почти 100 лет, она является фундаментом всей остальной физики, там поставлены сотни разнообразных экспериментов, высказано и проверено не меньше гипотез. Свежесть взгляда в физике, конечно, нужна, но не обращаться за ней нужно не в детский сад ясельную группу, в которой пребывает автор.
        Но это так, общие слова, а насчёт сего опуса, скажите честно, вы вообще поняли, что хотел автор? Сначала скопипастил из какой-то статьи, потом вывалил свои «рассуждения», представляющие малосвязный набор наукообразных слов, которые он когда-то где-то слышал, но смысла не знает. Если бы знал, не стал бы их ставить в таком порядке, ибо они не так стыкуются. И завершил всё это гениальным «нужно только поставить правильный эксперимент.» Какой именно — скромно умолчал, то есть проверить его «теорию» нельзя. С логикой у него, кстати, тоже не всё в порядке. Вот например: «Фотон не имеет ни прошлого, ни будущего. Для него есть только текущий момент, в котором оно существует», но «Я считаю, что фотон безусловно может предсказывать свое будущее»


        1. bull1251
          11.11.2016 17:00

          Я же вам ответил выше.

          Ну вообще то я закончил физико-математический факультет. И в какой то степени разбираюсь в данной области. К тому же нельзя сказать, что построил такую теорию за пару дней. В течении многих лет я задумывался над этими вопросами, с каждым разом все ближе и ближе подбираясь к сути, но не мог уловить самой основы, не мог представить в себе как выглядит пространство фотона. А после прочтения указанной статьи я с большей силой заинтересовался этими вопросами и решил постараться выстроить полную картину, собрав в едино все имеющиеся свои идеи. В процессе детального анализа я вдруг осознал как нужно смотреть, чтобы представить пространство фотона.
          Имеются ли у вас основание обвинять других в некомпетентности? Для подобных обвинений, я полагаю, вы должны лучше разбираться в данной области. В таком случае, не могли бы вы конструктивно объяснить в чем заключаются ошибки в моих суждениях?
          Зачем вы троллите? Просто конструктивно объясните в чем заключаются ошибки в моих суждениях, не переходя на личности.


          1. rerf2010rerf
            11.11.2016 17:14
            +1

            Объяснил https://geektimes.ru/post/282448/#comment_9687404
            А насчёт физико-математического факультета. Знаете, я тоже мехмат закончил, и какие там личности порой попадаются и как они учатся прекрасно знаю. Так что — не впечатлили.


        1. Olorin111
          12.11.2016 02:28

          стыдно сказать, но момент копипаста упустил. Переосмыслил. Спасибо.


    1. DoNotPanic
      11.11.2016 16:50

      А в чём проблема? Просто в реальности для «всей это учёной братии» идеи, высказанные в посте не являются чем-то новым, в конечном итоге к проверке подобных возможностей всё и идёт. Но это не говорит о том, что мысли, высказанные автором, плохи.
      Да и область такая… Довольно нестандартная в плане интерпретаций. Насколько могу судить, даже в учёной среде мнений насчёт того, как это работает, много, они не едины. Гипотетически даже человек, с темой слабо знакомый, как раз тут может оказаться сторонником верного мнения (или почти верного, но немного упрощённого) или даже предложить что-то своё (как вариант — не уникальное, но малопопулярное).


  1. gsaw
    10.11.2016 14:39
    -1

    Тема конечно кажется мне ересью, но написанно все доходчиво. Честно говоря первый раз прочитал понятное объяснение, почему время замедляется.


  1. norlin
    10.11.2016 14:51
    +2

    По-поводу замедления времени и т.д. – лично мне очень нравится гипотеза о том, что любой объект всегда движется со скоростью, строго равной скорости света. В 4-мерном пространстве. Т.е., скорость – это 4D-вектор неизменный по модулю, но с изменяемым направлением.

    Соответственно, чем больше проекция этого вектора на 3D-пространство, тем меньше остаётся на ось времени. И наоборот. А фотоны движутся строго перпендикулярно оси времени, т.е. проекция на ось времени равно нулю.


    1. wild_one
      10.11.2016 17:05

      А откуда мы можем быть уверены, что вектор вообще не 4+n-мерный? (с моей дилетантской точки зрения)? Ведь в пространстве могут быть и «свернутые» измерения.
      Кстати, а что по поводу квантования времени? Планковская длина есть, а «квант» времени?


      1. norlin
        10.11.2016 17:55

        Могут, а могут и не быть. Пока что известно только про 4, а так-то вообще что угодно можно предположить.


      1. 3al
        10.11.2016 20:06

        Планковская длина есть


        Но она не квант пространства и не обладает особым физическим смыслом.


  1. vinograd19
    10.11.2016 15:11

    1. Ловя один из фотонов (делая измерение), не имея инфы о другом фотоне, вы не можете утверждать, что именно ваше измерение редуцировало волновую функцию.
    2. Делая измерение, вы не можете влиять на его результат — он вероятностный. Поэтому никакой заранее подготовленный бит никуда передать нельзя.

    Задам автору его же вопрос.

    каким образом следствие (которое произошло в прошлом) может формировать причину (которое должно произойти в будущем) ?



  1. muhaa
    10.11.2016 15:42
    +1

    Просто прочитайте книгу Сасскинда «Квантовая механика. Теоретический минимум». Основы квантовой механики там разжеваны до такой степени, что понять их можно с начальной подготовкой инженерного вуза. На осознание скорее всего понадобится несколько месяцев. После этого можете пробежаться по начальным главам учебников, типа Ландау-Лифшица 3.
    После этого посмотрите на свою идею снова.
    У меня тоже когда-то сходной природы «идеи». Я сделал как советую Вам. Идеи не исчезли совсем, одни ушли, другие пришли. Для проверки этих других снова надо погружаться в учебники и так далее.


  1. RigelNM
    10.11.2016 16:00
    +1

    Разработка машины времени совершенно безрисковая инвестиция. Нужно в договоре добавить строчку, что первая экспедиция должна быть назначена на время за минуту до подписания договора. Если никто не придет, то можно не подписывать.

    В моей логике есть парадокс?


    1. rerf2010rerf
      10.11.2016 16:49

      Бабло получили, попили, договор всё равно не подписали, соответственно ничего не построили…


    1. QDeathNick
      10.11.2016 16:53

      Ну, если это всё ещё подписать ЭЦП, чтобы пришёл человек, вы уже после этого генерируете подпись, и сличаете с той, что он принёс. :)
      После этого можно было бы и подписать, только зачем? Ведь если вы не подпишите, всё равно всё уже будет создано, ведь он пришёл.


      1. RigelNM
        10.11.2016 17:17

        Но ведь это будет равносильно убийству своего дедушки в прошлом. Вот он и парадокс.


        1. Rampages
          10.11.2016 18:14

          Допустим убили вы дедушку, но это произошло в прошлом другой ветки событий и вашу историю/существование, оно не должно затрагивать.
          На мой взгляд должно существовать бесконечное количество альтернативных вариантов событий начиная с самого начала бытия.


          1. RigelNM
            10.11.2016 18:17

            Это понятно, но разве тебе хочется жить в мире где прошлое непоправимо? :)


      1. Stalker_RED
        11.11.2016 00:06

        Раз уж он из будущего будет идти, пусть захватит с собой копию чертежей машины времени и биржевую сводку.


    1. bull1251
      11.11.2016 19:11

      >Нужно в договоре добавить строчку, что первая экспедиция должна быть назначена на время за минуту до подписания договора.

      Не возможно вернуться в момент прошлого, где самой машины времени не существует.


  1. XoJIoD
    10.11.2016 16:48
    -1

    Видимо книга Брайана Грина «Ткань космоса» (а судя по иллюстрациям автор вдохновлялся ей) каждого читателя подталкивает на эту идею) Я, в своё время, тоже после её прочтения описал эту идею, правда не так развернуто, Арутюнову Константину Юрьевичу из ВШЭ. Но толи мы друг друга не поняли, толи он уже устал отвечать на подобные вопросы, потому что в конечном итоге он перестал мне отвечать) Так что вместе с автором жду опровержения от более подкованных в матчасти товарищей


  1. vkegdzoy
    10.11.2016 18:09

    Времени нет.


    1. Rampages
      10.11.2016 18:20
      -3

      А если найду? ?(????)


    1. savostin
      10.11.2016 23:10

      Ещё скажите «Время и стекло»


  1. maslyaev
    10.11.2016 19:40

    Тут выше уже написали про то, что получать результаты завтрашних лотерей через такую чудо-машинку, похоже, никак не получится. Ну и ладно. Будем зарабатывать деньги честным трудом.

    ИМХО, главный дефект всех рассуждений о путешествиях во времени (в данном случае — о передаче данных из будущего в настоящее) в том, что мы привыкли рассматривать время как некоторое дополнительное пространственное измерение. А это не так. Даже пространство-время Минковского — это всего лишь визуализация, позволяющая понять некоторые аспекты происходящего (например, парадокс близнецов). Не нужно забывать о том, что пространственные измерения качественно отличаются от временнОго. В пространственных измерениях мы можем вернуться туда, где были (положить пиво в холодильник, а потом его оттуда забрать), а с временнЫм измерением такой фокус не проходит. Качественное отличие.

    Даже с самым обычным движением по времени (строго вперёд, вместе со всем остальным миром) тоже, если вдуматься, ерунда получается. Движение — это всегда изменение чего-то со временем. «Изменение времени со временем со скоростью строго 1 сек/сек» — попахивает каким-то чудовищным абсурдом. Как-то так по всему выходит, что всё течёт, всё изменяется — сменяется день и ночь, времена года, автомобили ездят, самолёты летают, дети растут — но существенным условием всего этого является то, что САМО время должно быть абсолютно статичной штукой, намертво зафиксированной в положении «сейчас».

    А если «сейчас» зафиксировано, то ни о каких путешествиях во времени говорить в принципе нельзя. Перемещаться можно только тогда, когда есть хоть какое-то пространство для манёвра. А в данном случае не существует самого того пространства, по которому мы собираемся перемещаться. Извините, если кого обломал.


    1. vinograd19
      10.11.2016 19:57

      А в данном случае не существует самого того пространства, по которому мы собираемся перемещаться

      Существует. Это обычное пространство. Примеры использования такого путешествия во времени: наблюдение света отдаленных галактик, годичные кольца. В обоих случаях есть значения dt/dx, то есть той самой скорости перемещения во времени, о которой вы писали.


      1. maslyaev
        10.11.2016 20:44

        Наблюдение всегда происходит в «сейчас». Имея некоторые (в целом верные) представления о том, как устроен мир вокруг нас, мы делаем выводы о том, где и когда происходили наблюдаемые события. То есть определяем и пространственную, и временную координату. Но только если пространственная координата — это реальность (теоретически мы можем туда отправиться), то временная — условность. Очень полезная, но условность.

        Пример. «А.С.Пушкин родился в Москве в Немецкой слободе в 1799 году». Можно отправиться в Москву, приехать на станцию Бауманская и побывать в том самом месте, где родился Пушкин. А в том самом времени побывать нельзя. А почему нельзя? Объяснений может быть два:
        1. Потому что ещё не изобрели машину времени, которая нас туда могла бы переместить.
        2. Потому что в нашей грешной физической реальности не существует (обратите внимание, глагол в настоящем времени) никакого такого места «1799 год». Когда календарик показывал 1799 год, это была та самая точка «сейчас», в которой и мы с вами сейчас с удовольствием обитаем.

        Мне больше нравится второе объяснение. В нём меньше путаницы и логических косяков. Хотя, конечно, с наглядностью и привычностью у него большие проблемы.

        Событий, которые происходили в прошлом, уже нет. Но есть в нашем «сейчас» их последствия. Чтобы сориентироваться в том, что мы сейчас имеем (например, стихи Пушкина), нам удобно к трём пространственным измерениям добавить четвёртое, тем самым свести воедино «где» и «когда», и получить возможность оперировать динамикой. И то, что прошлых событий уже нет (есть только их последствия в «сейчас»), а будущих ещё нет (есть только наши намерения, которые тоже в «сейчас»), нас не должно расстраивать.

        Временная шкала, визуализирующая прошлое, настоящее и будущее — чрезвычайно полезная абстракция. Отказываться от неё ни в коем случае не нужно. Но нужно понимать, что это всего лишь абстракция, а никак не реально существующая какая-то там (где?) ось.


        1. vkegdzoy
          11.11.2016 07:18

          Ну вы даёте! Вы имеете верные представления о том как устроен мир? Начните с темной материи и энергии, которые невозможно зафиксировать точнейшими из приборов нами созданных.
          А время только у вас в голове, вы сравниваете моментальные события записанные раньше и позже. Так же о существовании Пушкина вы узнали о статье в википедии и о существовании фотона бы узнали только поймав его. Материальный объект единственное подтверждение существования чего-либо, не имя его вы не имеете и понятия времени, поскольку не можете сравнить двух объектов.
          Отправится во времени вы можете только переместив ВСЕ атомы во Вселенной на нужные вам места и никак иначе, для этого понадобится вся энергия Вселенной и её полное уничтожение.


          1. saboteur_kiev
            11.11.2016 13:57

            Начните с того, что если темную энергию и материю нельзя зафиксировать, то их просто несуществует. Начните с того, что эта темная материя/энергия — это не истина, а всего лишь гипотеза, которая была придумана, чтобы объяснить расхождение между тем что мы наблюдаем в движении планет и звезд, и как мы это рассчитываем. Вдобавок уже пару дней назад была выдвинута другая гипотеза, которая эти расхождения объясняет лучше, без всяких темных материй.

            Время — это не измерение. Время — это просто причинно-следственный процесс взаимодействия материи и энергии.
            Нельзя взять один объект, и нарушая эту связь переместить его по времени куда-либо — само пространство связано временем именно последовательно.

            Если даже гипотетически вы сможете переместить все атомы во вселенной на нужные вам места, мы не отправимся куда-либо во времени. Мы просто воссоздадим копию события в прошлом, но это не будет тоже самое событие.
            А материальный объект это не единственное подтверждение. Множество объектов излучают различные виды энергий, по которым мы можем предположить существование чего-либо весьма достоверно, без «пощупать».


            1. rerf2010rerf
              11.11.2016 14:02
              +1

              Вдобавок уже пару дней назад была выдвинута другая гипотеза, которая эти расхождения объясняет лучше, без всяких темных материй.

              Гипотез этих было выдвинуто уже десятки, если не сотни, и не только пару дней назад, а в течение десятилетий — ровно с момента обнаружения сих феноменов. Проблема всех этих гипотез в том, что, объясняя одно, они ломают другое, противоречат другим данным. Так что гипотеза тёмной материи самая «экономичная» из них.


            1. maslyaev
              11.11.2016 14:52

              Вдобавок уже пару дней назад была выдвинута другая гипотеза, которая эти расхождения объясняет лучше, без всяких темных материй

              На редкость корявая гипотеза. Моё возражение против неё тут: https://geektimes.ru/post/282432/#comment_9684832

              Время — это просто причинно-следственный процесс взаимодействия материи и энергии

              Понятие «время» завязано в очень тесный клубок с понятием «причинность». Разбудите любого среди ночи и спросите, что он знает о причинах и следствиях, и он, не приходя в сознание, сразу ответит, что следствие не может быть раньше причины. Внутри понятия «причинно-следственная связь» уже сидит неявно подразумеваемое понятие «время». Такие дела.


          1. maslyaev
            11.11.2016 14:13

            Вы имеете верные представления о том как устроен мир? Начните с темной материи...

            Я разве где-нибудь писал о том, что мы владеем абсолютной истиной во всей её полноте? Говоря о В ЦЕЛОМ верных представлениях, я имею в виду лишь то, что наших пониманий достаточно для того, чтобы хоть как-то более-менее сносно обслуживать себя в бытовом плане и не убиваться насмерть при любой попытке хоть что-нибудь предпринять.

            А время только у вас в голове

            В голове у меня мозги. А время если там внутри и есть, то точно так же и ровно столько же, сколько и во всех остальных предметах.

            о существовании Пушкина вы узнали о статье в википедии

            Вы мне льстите. Я не настолько молод ;)

            Материальный объект единственное подтверждение существования чего-либо

            Материальный объект — подтверждение существования только его самого. А всё остальное — наши домыслы. Мы ловко умеем строить эти домыслы (в конце концов, именно для этого у нас и есть желеобразная масса между ушами), но всё же не нужно забывать, что это всё лишь домыслы.

            Отправится во времени вы можете только переместив ВСЕ атомы во Вселенной на нужные вам места

            Давайте уж сразу волновые функции разложим как было…
            Проще надо быть, проще. Для возврата в прошлое у нас модно использовать гуманитарные технологии.


        1. astraleuro
          11.11.2016 15:52

          Мне нравится Ваша идея (хотя хотелось бы конечно чтобы путешествия во времени были возможными)
          Для себя представляю пространство-время в виде лифта, где кабина — это все пространство во вселенной, а шахта — это временная ось, всё пространство движется во времени и существует только в данный момент, теоретически, конечно, можно отправиться в прошлое (выбраться из кабины и поползти по шахте вниз), но пространства там (тогда?) уже не существует.


    1. bull1251
      10.11.2016 20:16

      Интересная точка зрения. Но как вы сказали Движение — это всегда изменение чего-то со временем. И даже если мы не сможем путешествовать во времени, мы ведь можем искать следы этих изменений.

      По поводу того, что САМО время должно быть абсолютно статичной, думаю нельзя исключать такой вариант и тоже требует рассмотрения. Возможно мы ощущаем течение времени только потому, что мыслительные процессы неразрывно связаны с биологическими процессами и чувство времени является неизбежным продуктом собственного сознания. Ведь остановка всех процессов организма приведет к тому, что мы перестанем что либо осознавать.


      1. maslyaev
        10.11.2016 20:56

        И даже если мы не сможем путешествовать во времени, мы ведь можем искать следы этих изменений.

        Собственно, так мы всю жизнь и делаем. И очень в этом поднаторели ;)

        Такая точка зрения на время — вовсе не моё изобретение. В первый раз я с такими рассуждениями столкнулся в книжке Дэвида Дойча «Структура реальности». Но вообще, в целом похожие рассуждения о времени есть у Канта в КЧР. Эйнштейн, кстати, эту книжку зело уважал.

        На физиологию я бы особо не завязывался. Очень большой риск нарваться на логическую петлю.


  1. alexhott
    10.11.2016 19:59

    Кроме теорий ничем не подкрепленных для понятия Время пока ничего нет.
    Моя теория: время — это выдуманное человеком понятие для измерения промежутков между прошлым и будущим.
    Если проводить аналогию с чемто более понятным, то время это как Температура.
    Может быть 0 — то есть я допускаю возможность остановить время
    Но по аналогии с температурой не может быть ниже 0.
    По этой теории в будущее можно путешествовать, а в пролое не получится.

    Если заморозить человека — мы локально для него остановим время, через 20 лет разморозим — и он путешественник во времени.


  1. ShabanovYT
    10.11.2016 23:05
    -1

    Работаю над установкой, которая будет перемещать 1 мг золота на 10-8 сек в будущее. Так как в настоящем образец никуда исчезнуть не может, в 3 м от установки должен появиться еще один 1 мг. Вроде так.
    Эксперименты довольно опасны из-за того, что закон сохранения энергии должен соблюдаться. Например, вокруг установки в радиусе 100 м все может охладиться до температуры абсолютного нуля, неожиданно. Нужны, короче, добровольцы-лаборанты. Родина их не забудет.


  1. Delics
    10.11.2016 23:20

    > Но что будет, если мы решим действовать по другому, вопреки тому, что отображается
    > на экране?

    Этот парадокс легко разрешается в работах астрофизика Игоря Новикова — решив нарушить «послание из будущего» вы тем самым мгновенно создадите усредненную ситуацию между разными выборами решения.

    И решение всегда будет одно.


    1. bull1251
      10.11.2016 23:54

      Я поэтому указал в тексте:
      > мы изначально должны были видеть на экране смесь интерференций и двух полос. А значит, мы изначально не могли бы определиться с противоположным выбором, что снова приводит нас к парадоксу…

      Если мы не можем определиться с выбором, то у нас возникнет время подумать — продолжать или не продолжать эксперимент. Причем вероятность второго на мой взгляд превышает первое. Если мы всё же решим продолжить, то возникает выбор — наблюдать или не наблюдать за фотоном, что еще больше увеличивает вероятность второго.
      Т.е. вероятность двух событий не равно между собой, а это создает возможность противоположного выбора. Возможность противоположного выбора пытается усреднить картину, что снова приводит нас вышеуказанному изменению вероятности с нашей стороны. Так что же должно отображаться на экране? Парадокс.


      1. Delics
        11.11.2016 21:04

        Сразу или потом — неважно. Выбор всё-равно усреднится.


  1. napname
    11.11.2016 00:02
    -1

    Публикация заставила задуматься и соотнести некоторые моменты.
    Фотон — квантовая частица.
    Если для неё нет понятия времени, то она содержит в себе все возможные варианты развития событий одновременно.
    Из квантовой механики вспоминается что квантовая частица имеет бесконечное множество состояний, в момент измерения можно выбрать только одно из этих состояний.
    В случае с щелями получаем что фотон имеет информацию о всех возможных вариантах прохождения через них, а измерением мы отмечаем одно.

    Ну и напоследок крутится в голове мысль — а не получится ли создать щелевой квантовый компьютер?


    1. bull1251
      11.11.2016 23:09

      Идея интересная. Но если фотон имеет форму вытянутой струны, то он не может быть вытянут во все стороны. Скорее волна времени может распространиться во все стороны, затрагивая все соседние фотоны на пути (как круги на воде). Но волна может пересекать наблюдателя только в одной точке, в котором он может наблюдать фотон.

      При прохождении через две щели волна времени может разделяться, образуя две волны (соответственно две линии времени). Пересечение этих волн образует интерференционную картину вероятности на линии времени наблюдателя. Но для самого наблюдателя существует только одна линия времени. Он не может одновременно переживать два разных события, взаимоисключающих друг друга. Соответственно, наблюдатель может видеть фотон только в одной точке пространства.

      Теперь попробуем разобраться почему на экране возникает интерференционная картина. Из одного фотона получить какую либо картину невозможно. Чтобы на экране образовалась видимая картина выпускают множество фотонов по очереди. Каждый фотон может попадать в разные точки экрана, образуя картину интерференции. Но если на пути к экрану мы зафиксируем откуда прошел фотон, то снова исключим возможность одновременного существования двух взаимоисключающих событий. Проще говоря, процесс наблюдения приводит к слиянию нескольких линий времени в одну линию времени, в котором пребывает наблюдатель.

      Но это конечно только мои предположения. Чтобы построить теорию нужно развивать идею, делать всесторонние анализы и расчеты, выводить закономерности. А для этого нужны сторонники в научных кругах, чтобы занимались этим в рамках научной работы. Я же просто сисадмин, и у меня другие интересы по жизни))


  1. Fib0na4i
    11.11.2016 01:41
    +5

    Не мог не прокомментировать, физик по образованию. По этому поводу хорошо высказался Пол Парсонс — редактор книги «Научные теории за 30 секунд».



    Цитата
    У каждого из нас есть своя излюбленная теория.
    Я это знаю не понаслышке. Когда я работал редактором ежемесячного
    научно-популярного журнала ВВС «Focus», то каждый день получал по
    электронной почте несколько таких теорий. Это были послания от чита-
    телей, утверждавших, что они раскрыли тайну черных дыр, параллельных
    миров или Большого взрыва, что они установили, откуда взялась жизнь,
    и объединили законы физики частиц. Я отвечал, благодарил за теории и
    просил прислать подтверждающие их математические выкладки. Ни
    один читатель ни разу этого не сделал.
    В этом и заключается разница между «теориями», которые мы обсужда-
    ем в повседневной болтовне (основываясь на наших слабых представ-
    лениях о предмете и весьма приблизительных расчетах), и теориями,
    старательно выстроенными учеными.




    Рекомендую автору поста к прочтению кроме вышеуказанной книжки, еще:
    Мичио Каку “Гиперпространство”
    Билл Брайсон “Краткая история почти всего на свете”

    Для справки — кванты нам преподавали на протяжении двух семестров два
    раза в неделю. Одна пара — теория, одна пара — практика.

    Завидую автору поста, «пару дней» подумал и машина времени практически готова.


    1. rerf2010rerf
      11.11.2016 14:36
      +1

      Я не физик, но мнение о книжках вроде упомянутого «Гиперпространства» имею. На мой взгляд, от них больше вреда, чем пользы. Поскольку нормально объяснить квантовую физику, ОТО, да даже и СТО, не говоря уж о более современных вещах без математики невозможно, то авторы начинают городить такой дикий многоэтажный огород отдалённых аналогий, что разглядеть за ними физическую суть невозможно. В итоге, после чтения таких книжек возникает дикая каша в голове, без какого бы то ни было реального понимания, зато уж иллюзия понимания и причастности к некой «понимающей элите» возникает несомненно. А главное, возникает иллюзия, что всё это научно-популярное словоблудие без единой формулы — это и есть настоящая физика, а значит и ощущение, что «я тоже так могу, щас так же накидаю кучу наукообразных слов в одну кучу, и вот вам ещё десяток теорий». Отсюда и такие вот теории, как в этом посте.
      Знаю всё это на своём опыте, тоже когда-то начитался популярных книжек и думал, что что-то понял. Потом отучился 4 года на мехмате и понял на физике, что всё, что я понимал, вернее думал, что понимал, о квантовой механике, было полным бредом. До других областей физики так и не добрался. По моему, если уж пытаться объяснять основы современной физики, то делать это с математикой, хотя бы на самом простом из необходимых для нормального понимания уровне, но не проще. Или не пытаться вообще.
      Например, тут уже упоминали книги Сасскинда «Теоретический минимум» — вот это, на мой взгляд, хорошая научно-популярная книга, которая действительно поможет разобраться. А объяснения свёрнутых измерений через муравьёв на шланге и подобные нужно отправлять в мусорку не читая, чтобы мозг не засорять.


  1. ofmetal
    11.11.2016 11:12

    1. Перемещение информации из будущего в прошлое тождественно перемещению со скоростью выше скорости света.
    2. Следствие из второго постулата СТО предусматривает существование предельной скорости, скорости света, быстрее которой информация не может быть передана.
    => 3. Передача информации в прошлое противоречит СТО.


  1. Fenyx_dml
    11.11.2016 12:34

    «Возникает такой парадокс, что установка сама предопределяет будущее.»

    Вы читали «Дюну»? Там есть такая идея, что оракул скрывает себя и тех кто находится под его влиянием от других оракулов… Меня как-то зацепила эта идея, я её обдумывал и пришел к выводу, что наличие способности к предсказанию у любого разумного существа автоматически приводит к изменению будущего. А раз так, для оракула будущее предопределено. Он сам его пораждает из первоначального хаоса путем своего свободного выбора. Это трудно объяснить словами, но получается так, что видя бесконечное число вариантов развития будущего и причинно-следственные связи, приводящие к этим вариантом он просто ОБЯЗАН выбрать один из них, причем уход от выбора невозможен. Он либо делает то лтбо это, либо ничего — любой вариант приводит к совершенно определенным событиям, которые он (оракул) изначально знает! Понимаете что это означает? Если такая установка будет создана, то последствия будут такими, которые никто еще даже себе не представил! Ну, конечно, простая установка, типа «скажи мне что выпадет — орел или решка» и последствия вызовет скромнее. Ну как скромнее… Локальный конец цивилизации на планете Земя вполне можно устроить, причем непреднамеренно. Я не шучу и на вскидку мог бы привести несколько таких сценариев, вспомните только что закончившуюся предвыборную кампанию в США. Что если кто-то сможет заранее узнать — победит претендент 0 или 1??? Так что автор, зря ты это опубликовал… Хотя может это и было предопределено… Может потому мы и не видим сигналов инопланетян — все кто дошел до определенного этапа изобрели машину-оракул и накрылись медным тазом…


  1. fzzr
    11.11.2016 15:51

    Мне кажется я вижу логическую ошибку в части про вертикально выпущенный фотон между двух параллельных горизонтальных зеркал.


    При достижении скорости света рассматриваемая траектория движения будет выпрямлена в одну линию

    Две машины едут перпендикулярно одна относительно другой, скорости их равны. Траектория одной относительно другой не будет перпендикулярна, ведь объекты сближаются (или отдаляются) один относительно другого с одной скоростью. Траектория будет диагональная, 45 градусов.


    1. bull1251
      12.11.2016 01:23
      -1

      Если траектория будет диагональная, то луч света будет пересекать пространство быстрее скорости света, по той причине что луч будет преодолевает большее расстояние двигаясь также со скоростью света по горизонтали. Это противоречит теории относительности.

      Но вы правильно заметили насчет угла 45 градусов. Именно на этот угол будет развернута линия времени луча, относительно нашего времени, что приведет к остановке времени для луча. Наше пространство свернется по направлению движения луча, а траектория движения луча развернется в пространство самого луча. Время станет частью пространственного измерения луча. Думаю не трудно заметить, что для таких расчетов уже не годятся формулы классической механики.


  1. AntkliKruin
    12.11.2016 13:52
    +1

    Прежде чем отправляться на поиски читателя, нужно научиться писать. Концентрация косноязычия и «падежов» на кв. см. текста превышает все нормы.
    С самоуважением, дорогой читатель.


  1. EvgK
    14.11.2016 14:48
    +1

    Не знаю, писали в комментариях уже или нет, но в эксперименте с отложенным выбором у вас сделано неверное предположение. Вы предполагаете, что после установки даун-конвертеров вы получите интерференционную картину в зависимости от того, наблюдаете ли вы за холостыми фотонами. Однако это не так — после установки даун-конвертеров на экране никогда не будет интерференционной картины, что бы вы ни делали с холостыми фотонами.


    1. bull1251
      19.11.2016 03:03
      -1

      На самом деле это только начало эксперимента, позволяющий понять суть происходящего. Поэтому в тексте я указал:

      В качестве примера рассмотрим эксперимент с отложенным выбором, который был предложен Скалли и Дрюлем:
      В дальнейшем установка усложняется. Причем дальнейшее действия будут во многом отличаться от предложенного эксперимента Скалли и Дрюля. Я не стал описывать всех деталей предполагаемого мной эксперимента, оставив для читателя возможность самостоятельно решить эту задачу. А чтобы получить интерференционную картину, в последующем необходимо будет стереть информацию о том, через какой путь прошли фотоны. Например, направив холостые фотоны обеих направлений в один даун-конвертор, дающий на выходе «50 на 50» вероятности того, через какой путь прошли фотоны.

      Я читал ваши комментарии, и в особенности хотелось бы отметить данный текст:
      Если фотон попал в D1 или D2 — информация о пути «стерта». Если мы удалим маркеры со всех фотонов, они все попадут в D1 или D2. Но важный момент в том, что мы получим ДВЕ интерференционные картины (для фотонов из D1 и D2 соответственно), причем пики одной совпадают с впадинами другой. Соответственно эти две картины в сумме дают обычное хаотическое распределение (которое мы и видим на экране).
      Дело в том, что ДВЕ интерференционные картины при наложении дают хаотическую картину, только когда используются два лазерных луча разного спектра (красного и синего). Предположение верное, но только в рамках указанного вами эксперимента. А когда используется один лазер, такого смещения не происходит.


      1. EvgK
        19.11.2016 10:26

        Я специально узнавал у двух разных профессоров про данную конкретную ситуацию, и оба подтвердили этот факт. После прохождения даун конвертеров, новые фотоны не когерентны (потому, что они образуются в разных случайных точках внутри кристалла даун конвертера, а не в одной конкретной точке) и не образуют интеренференционной картины.


        1. bull1251
          19.11.2016 16:29

          Это хороший аргумент. Давайте попробуем разобраться в этом вопросе. Допустим, что новые фотоны образованные в даун-конвертерах не когерентны. В таком случае следует вывод, что интеренференционная картина не может формироваться ни при каких условиях. Не зависимо от того наблюдают или нет за холостыми фотонами, интеренференционная картина будет разрушена отсутствием когерентности.

          Теперь предлагаю рассмотреть полное описание эксперимента Скалли и Дрюля http://www.physic-in-web.ru/study-119-12.html (с 12 по 15 стр.). Приведу не большой отрывок из описания:

          Итак, для сигнальных фотонов, холостые партнёры которых обнаружены детектором 1 или 4, мы имеем информацию о выбранном пути, но для сигнальных фотонов, холостые партнёры которых обнаружены детектором 2 или 3, информация о выборе пути стёрта (рис. 7.5б).

          Означает ли это стирание части информации о выборе пути — хотя мы ничего не делаем с сигнальными фотонами непосредственно — что интерференционные эффекты восстанавливаются? Это действительно так, но только для тех сигнальных фотонов, чьи холостые партнёры попали в детектор 2 или детектор 3. Именно, места попадания всех сигнальных фотонов на экран будут давать картинку, похожую на данные для рис. 7.5а, не показывающего даже самого слабого намёка на интерференционную картину, что характерно для фотонов, которые идут либо одним, либо другим путём. Но если мы рассмотрим лишь подмножество результирующих точек — например, от тех сигнальных фотонов, для которых холостые фотоны попали в детектор 2, — то это подмножество точек будет давать интерференционную картину! imageimage
          О каком восстановление интерференционной картины идет речь, если фотоны образованные в даун-конвертерах не когерентны? Даже если рассматривать только подмножество результирующих точек из детектора 2, отсутствие когерентности должно разрушить интерференционную картину!

          Из всего этого следует вывод, что либо ошибочно предположение о не когерентности фотонов, либо экспериментаторы нам врут. Но зачем описывать в научных книгах заведомо ложные результаты экспериментов? Ведь если верить описанию, то результаты экспериментов были положительны:
          Скалли и Дрюль предложили этот эксперимент, потому что квантово-механические вычисления убедили их, что это будет работать. Так и произошло. Как и обычно с квантовой механикой, головоломка не противопоставила теорию и эксперимент.
          Я же считаю, что когерентность фотонов сохраняется даже после прохождения даун-конвертеров. Потому что разделившиеся на выходе фотоны были порождены одним и тем же фотоном на входе. Даже если они образуются в разных точках, они наследуют все свойства изначального фотона. Доказательством этого можно рассматривать успешно проведенные эксперименты.


          1. EvgK
            19.11.2016 16:55

            Но ведь это совсем другой эксперимент. И они действительно рассматривают лишь подмножество точек (сигнальных фотонов), поскольку лишь подмножество может (в результате определенных манипуляций) дать интеренференционную картину, и мы узнаем об этом только после изучения холостых фотонов. Вы же утверждаете что можно получить интерференцию всех сигнальных фотонов, в завистмости от будущих действий с холостыми.


            1. bull1251
              19.11.2016 17:18

              И они действительно рассматривают лишь подмножество точек (сигнальных фотонов), поскольку лишь подмножество может (в результате определенных манипуляций) дать интеренференционную картину
              По какой причине рассматриваемое подмножество точек может дать интеренференционную картину? Значит для данного подмножества фотоны должны быть когерентны. Но могут ли фотоны из одного подмножества быть когерентными, а из другого подмножества быть не когерентными? Такого в принципе быть не должно.

              и мы узнаем об этом только после изучения холостых фотонов. Вы же утверждаете что можно получить интерференцию всех сигнальных фотонов, в завистмости от будущих действий с холостыми.
              Посмотрите в предыдущем комментарии рисунок (б). Что нам мешает в место даун-конверторов А и Б поставить зеркала под углом 45°, тем самым исключив из эксперимента детекторы 1 и 4? В этом случае все холостые фотоны будут направлены в даун-конвертер С, стирающий информации о выборе пути. Значит все сигнальные фотоны сохранят квантовую неопределенность и будут формировать на экране интеренференционную картину.


            1. bull1251
              19.11.2016 18:25

              А если мы уберем эти зеркала (например, поднимем их вверх), то все холостые фотоны будут зафиксированы детекторами 1 и 4. Тогда на экране интерференция пропадет.


          1. EvgK
            19.11.2016 17:01

            Что касается когерентности, фотоны являются частью когерентного состояния, но это состояние включает в себя как сигнальные, так и холостые фотоны (они запутаны). Сами по себе сигнальные фотоны не являются когерентными. Кстати даже в вашей цитате написано, что для эксперимента на рис 7.5а нет даже слабого намека на интерференционную картину.


            1. bull1251
              19.11.2016 17:24

              О какой интерференции может идти речь, если холостые фотоны попадают на все 4 детектора? В этом случае на экране возникает смесь, из которого нельзя восстановить интерференцию. А чтобы получить интерференцию на экране, нужно исключить из эксперимента 1 и 4 детекторы, как я писал выше.